[obm-l] Re: [obm-l] Re: [obm-l] Re: [obm-l] cadeira de 3 pés

2023-01-23 Por tôpico Bernardo Freitas Paulo da Costa
On Mon, Jan 23, 2023 at 12:52 PM Claudio Buffara
 wrote:
>
> Obrigado, Wagner e Ponce:
>
> Eu tinha pensado em algo na linha do que o Ponce escreveu, ainda que em 
> certos casos patológicos (pelo menos de piso...) o terceiro pé pode não 
> encontrar apoio: imagine um piso com um pico fino em algum lugar (p.ex. a 
> superfície gerada pela revolução de z = 1/(1+(x^2+y^2)^20) em torno do eixo 
> z). Neste caso, precisaríamos de uma cadeira bem pequena, ou pelo menos com 
> as pontas dos pés bem próximas umas das outras.

Pois é, eu acho que tem uma questão interessante de estabilidade que é
a seguinte: se você pegar um banco de 3 pés e botar na encosta de uma
montanha (bem íngreme!), ele vai rolar ladeira abaixo.  Porque a
projeção do centro de gravidade cai fora do triângulo determinado pelo
contato com o solo (que define as normais que podem segurar a cadeira
em posição estática).  Uma cadeira de 4 pés só é bamba porque uma
pessoa não consegue ficar (infinitesimalmente) parada, e daí o centro
de gravidade oscila entre os triângulos formados por dois pares de
três apoios, por exemplo ABC e ACD.

Bernardo

-- 
Esta mensagem foi verificada pelo sistema de antiv�rus e
 acredita-se estar livre de perigo.


=
Instru��es para entrar na lista, sair da lista e usar a lista em
http://www.mat.puc-rio.br/~obmlistas/obm-l.html
=


[obm-l] Re: [obm-l] Re: [obm-l] Relação de girard

2020-11-07 Por tôpico Bernardo Freitas Paulo da Costa
On Thu, Nov 5, 2020 at 9:26 PM Artur Costa Steiner
 wrote:
>
> Para facilitar, suponhamos que o polinômio de grau n P seja mônico. Sejam 
> z_1, , z_n suas n raízes não necessariamente distintas. Para todo 
> complexo z, temos que
>
> P(z)  = ( z - z_1) (z - z_n)
>
> Desenvolvendo e aplicando o chamado produto de Stevin, vc tem as relações de 
> Girard.

Eu não conhecia o produto de Stevin, mas de forma geral quando você
usa "..." tem, muitas vezes, um argumento por indução que está
subentendido.  Pode ser que o produto de Stevin "faça a indução pra
você" (calculando os termos \sum \prod z_i que vão aparecer como
coeficientes dos monômios z^k), mas é "quase" como se você estivesse
empurrando a indução um andar abaixo ;-)


Israel: qual a demonstração por indução que você conhece?  E porque
você gostaria de outra??

Abraços,
-- 
Bernardo Freitas Paulo da Costa

=
Instru��es para entrar na lista, sair da lista e usar a lista em
http://www.mat.puc-rio.br/~obmlistas/obm-l.html
=


[obm-l] Re: [obm-l] Encontrar K mínimo

2020-05-23 Por tôpico Bernardo Freitas Paulo da Costa
On Sat, May 23, 2020 at 11:46 AM Jeferson Almir
 wrote:
>
> Amigos peço ajuda nesse problema, ou até algum resultado de grafos que 
> resolva.
>
> Terra Brasilis  possui 2021 cidades, e existem voos de ida e volta entre 
> algumas dessas  cidades de maneira que é possível chegar a qualquer outra 
> através de voos finitos. Encontre o menor inteiro positivo k tal que, 
> independente da configuração dos voos, é possível escolher k cidades de modo 
> que qualquer uma das 2021 cidades possui voo direto para alguma das cidades 
> marcadas.

Não entendi o que quer dizer "vôos finitos"...  Tem um enunciado mais
preciso, ou é isso aí?

E, para tentar ajudar: o que acontece se forem 2 cidades? 3?  Aliás,
uma pergunta, se o caso de 2 cidades for A - B, não tem "vôos de B
para B", então você teria que escolher as cidades (A,B) ?  Ou também
está faltando esse detalhe no enunciado?


Abraços,
-- 
Bernardo Freitas Paulo da Costa

-- 
Esta mensagem foi verificada pelo sistema de antiv�rus e
 acredita-se estar livre de perigo.


=
Instru��es para entrar na lista, sair da lista e usar a lista em
http://www.mat.puc-rio.br/~obmlistas/obm-l.html
=


Re: [obm-l] Zero Elevado a Zero

2020-02-15 Por tôpico Bernardo Freitas Paulo da Costa
On Sat, Feb 15, 2020 at 11:55 PM Luiz Antonio Rodrigues
 wrote:
>
> Olá, pessoal!
> Tudo bem?
> Eu aprendi que qualquer número elevado a zero é 1, mas com exceção do zero.
> Também aprendi que 0^0, assim como 0/0, representam indeterminações.
> Na minha calculadora científica, a operação 0^0 resulta em erro.
> Acontece que há pouco tempo eu vi num livro que era utilizado num conceituado 
> colégio de São Paulo que 0^0=1.
> Resolvi consultar a internet e descobri que essa discussão já existia no 
> tempo em que Euler era vivo...
> Um dos sites que eu consultei diz que existem três respostas possíveis, 
> dependendo do contexto:
> a) 0^0 é inexistente
> b) 0^0 é indeterminado
> c) 0^0=1
> Não fiquei convencido e acho que o item (b) é o correto, da forma como eu 
> aprendi.
> O que vocês pensam sobre isso?

O Ralph falou disso há pouco tempo na lista.  Eu ainda acho uma das
melhores respostas.  O início já é espetacular:
"""
[A] resposta eh "Decida como quiser, diga para todos como você
decidiu, e seja coerente. De preferência, escreva as coisas para
evitar a pergunta."
O problema eh a convenção
"""

Os detalhes e a argumentação você encontra em
https://www.mail-archive.com/obm-l@mat.puc-rio.br/msg56739.html

Só para responder um ponto sobre "é indeterminado" (e uma "firula de
linguagem"; leia a resposta do Ralph antes, vale mais a pena!).  Em
matemática, isso "não existe".  Ou é "não-definido", ou é definido.
Você pode dizer que 0^0 está "definido para representar a
indeterminação lim (a_n)^(b_n) quando ambos a_n e b_n tendem a zero"
(ou f(x) ^ g(x) quando ambas funções tendem a zero).  Você pode fazer
como o Ralph gosta (e eu também, digo logo) e definir 0^0 = 1.

Assim, a opção "a" corresponde a "não vou nem definir o que isso quer
dizer".  Como se você perguntasse "o que é "1+/"? ".  Não faz sentido,
é uma expressão inválida matematicamente.
A opção "b" seria "vou definir como símbolo (abreviação) deste
limite".  Observe que esta definição AO CONTRÁRIO da definição de 1+1,
não diz o quanto vale.  É tipo uma definição como "Z" sendo o símbolo
dos números inteiros.  Representa alguma coisa, mas não "vale" nada.
Enfim, a opção "c" é "vou definir", ESTENDENDO a operação de ^ para
que o domínio contenha o par (0,0).

Abraços,
-- 
Bernardo Freitas Paulo da Costa

-- 
Esta mensagem foi verificada pelo sistema de antiv�rus e
 acredita-se estar livre de perigo.


=
Instru��es para entrar na lista, sair da lista e usar a lista em
http://www.mat.puc-rio.br/~obmlistas/obm-l.html
=


[obm-l] Re: [obm-l] Re: [obm-l] Re: [obm-l] Re: [obm-l] Re: [obm-l] Re: [obm-l] Análise complexa - mostrar que f é sobrejetora

2020-02-11 Por tôpico Bernardo Freitas Paulo da Costa
On Mon, Feb 10, 2020 at 10:12 PM Artur Costa Steiner
 wrote:
>
> Em seg, 10 de fev de 2020 21:13, Pedro Angelo  
> escreveu:
>>
>> Aparentemente, errei hehe. Achei engraçada essa explicação: funções
>> holomorfas não-inteiras também têm "série de potências inteiras" em
>> torno de cada ponto. Por que só as inteiras levam o nome?
>
> Acho que inteira é no sentido de global, completa.

De fato.  A primeira evidência vem do próprio Picard: "Nous donnerons,
avec M. Weierstrass, le nom de fonctions entières d'une variable
complexe z aux fonctions uniformes et continues dans toute l'étendue
du plan; ce seront, par suite, des fonctions représentées par une
série, toujours convergente, ordonnée suivant les puissances
croissantes de la variable." (Mémoire sur les fonctions entières,
1880, justamente onde ele demonstra os "teoremas de Picard",
http://www.numdam.org/article/ASENS_1880_2_9__145_0.pdf).  Depois, tem
que ler em alemão alguém falando da história do Weierstrass (não achei
o livro / artigo onde ele usa esta notação pela primeira vez).  Eu
achei o Felix Klein, em
https://books.google.co.uk/books?id=XtunBgAAQBAJ=PA286=PA286=weierstrass+ganze+funktion=bl=E5OhNVM3WW=ACfU3U2ABjdB68sIwWisSpPCLJZf_8KdTQ=en=X=2ahUKEwjYteu1lsnnAhXioVwKHWlDAGYQ6AEwAHoECAcQAQ#v=onepage=weierstrass%20ganze%20funktion=false,
e de fato ele usa a mesma terminologia do Picard: ganzen Ebene (o
plano inteiro) e Potenzreihe (série de potências).

> Talvez seja uma tradução um tanto infeliz de entire function, do Inglês. No 
> Inglês, entire em nada lembra integer.

Em geral, eu chuto que um termo matemático usado antes do século XX
não vem do inglês; a França e a Alemanha eram os grandes centros
praticamente até a segunda guerra.

> Mas será que é possível provar o teorema sem invocar Picard?

Boa pergunta.  Será que o resultado é equivalente a Picard?  Acho
pouco provável, mas talvez valha a pena tentar...
--
Bernardo Freitas Paulo da Costa

-- 
Esta mensagem foi verificada pelo sistema de antiv�rus e
 acredita-se estar livre de perigo.


=
Instru��es para entrar na lista, sair da lista e usar a lista em
http://www.mat.puc-rio.br/~obmlistas/obm-l.html
=


[obm-l] Re: [obm-l] Re: [obm-l] Re: [obm-l] Análise complexa - mostrar que f é sobrejetora

2020-02-10 Por tôpico Bernardo Freitas Paulo da Costa
On Mon, Feb 10, 2020 at 8:16 PM Artur Costa Steiner
 wrote:
> O adjetivo inteira, em análise complexa,  não tem nada a ver com o que ele 
> sugere. Acho uma terminologia infeliz, mas é consagrada.

Um chute: em francês, o termo "série inteira" (por oposição a série
fracionária) se refere às séries de potências (inteiras) da variável z
(por oposição às "séries de Puiseux" onde há expoentes fracionários).
E as funções inteiras têm expansão, convergente, como série de
potências (inteiras) da variável z, f(z) = \sum_{n=0}^\infty a_n z^n.

Abraços,
-- 
Bernardo Freitas Paulo da Costa

-- 
Esta mensagem foi verificada pelo sistema de antiv�rus e
 acredita-se estar livre de perigo.


=
Instru��es para entrar na lista, sair da lista e usar a lista em
http://www.mat.puc-rio.br/~obmlistas/obm-l.html
=


Re: [obm-l] Volume de um Toroide

2020-01-30 Por tôpico Bernardo Freitas Paulo da Costa
Olá,

On Thu, Jan 30, 2020 at 11:21 AM Luiz Antonio Rodrigues
 wrote:
> Estou tentando resolver um problema há alguns dias e não estou conseguindo 
> chegar numa resposta correta.
> O problema é o seguinte:
>
> Qual a integral que representa o volume do disco
>
> ((x-b)^2)+y^2
> que gira em torno do eixo y?
> Considere 0 Primeiro eu preciso resolver usando dx e depois dy.
> Por fim, o problema pede o valor do volume em termos de a e b.

Fazer "primeiro com dx" de "depois com dy" é só "resolver a integral
dupla" primeiro em y, ou primeiro em x.  Também pode ser pensado
"fatiando" o seu círculo não em retângulos pequenos nas duas direções,
mas em apenas uma.  Mas eu acho isso mais complicado do que precisa (e
dá uns nomes estilosos tipo "cascas cilíndricas" e tal, mas não acho
que seja muito iluminador se você não entendeu estes quadradinhos...)

Abraços,
-- 
Bernardo Freitas Paulo da Costa

-- 
Esta mensagem foi verificada pelo sistema de antiv�rus e
 acredita-se estar livre de perigo.


=
Instru��es para entrar na lista, sair da lista e usar a lista em
http://www.mat.puc-rio.br/~obmlistas/obm-l.html
=


[obm-l] Re: [obm-l] Re: [obm-l] Re: [obm-l] Questão OBM - U

2020-01-23 Por tôpico Bernardo Freitas Paulo da Costa
On Thu, Jan 23, 2020 at 7:24 AM gilberto azevedo  wrote:
>> On Sat, Jan 11, 2020 at 11:24 AM gilberto azevedo  
>> wrote:
>> >
>> > Qual o ínfimo sobre todos os quadriláteros convexos com perímetro 8 da 
>> > soma dos comprimentos de suas diagonais ?
>
> Tentei com o retângulo e o quadrado, porém não obtive a resposta...  O 
> gabarito é 4.

Qual (ou quais?) retângulo(s) você testou??  Que resposta você obteve?
-- 
Bernardo Freitas Paulo da Costa

-- 
Esta mensagem foi verificada pelo sistema de antiv�rus e
 acredita-se estar livre de perigo.


=
Instru��es para entrar na lista, sair da lista e usar a lista em
http://www.mat.puc-rio.br/~obmlistas/obm-l.html
=


Re: [obm-l] Soma de Riemann

2020-01-15 Por tôpico Bernardo Freitas Paulo da Costa
On Tue, Jan 14, 2020 at 9:45 PM Claudio Buffara
 wrote:
> O artigo é esse aqui:
> https://epocanegocios.globo.com/Informacao/Dilemas/noticia/2014/12/elas-precisam-de-reengenharia.html
> É de 2014, mas ino que a situação não tenha mudado muito de lá pra cá.

Há algumas tentativas de mudança.  Uma delas, o recém-criado curso de
Engenharia Matemática da UFRJ.  Inspirado, em parte, da experiência de
intercâmbio com a Polytechnique e a ENSTA (tanto de professores como
de alunos), e buscando integrar a sólida formação em matemática e
ciências básicas com o maior centro de pesquisa em engenharia da
América Latina, a COPPE.

Para mais detalhes sobre o curso, confiram
https://sites.google.com/matematica.ufrj.br/aplicada/engenharia-matem%C3%A1tica
http://www.im.ufrj.br/index.php/pt/noticias-e-eventos/noticias/247-saiba-mais-sobre-o-novo-curso-engenharia-matematica

E, para quem quiser ler a proposta integral:
http://www.im.ufrj.br/images/documentos/projeto_engenhariamatematica.pdf

Abraços,
-- 
Bernardo Freitas Paulo da Costa

-- 
Esta mensagem foi verificada pelo sistema de antiv�rus e
 acredita-se estar livre de perigo.


=
Instru��es para entrar na lista, sair da lista e usar a lista em
http://www.mat.puc-rio.br/~obmlistas/obm-l.html
=


[obm-l] Re: [obm-l] Questão OBM - U

2020-01-11 Por tôpico Bernardo Freitas Paulo da Costa
On Sat, Jan 11, 2020 at 11:24 AM gilberto azevedo  wrote:
>
> Qual o ínfimo sobre todos os quadriláteros convexos com perímetro 8 da soma 
> dos comprimentos de suas diagonais ?

Quais são os quadriláteros que você tentaria?
-- 
Bernardo Freitas Paulo da Costa

-- 
Esta mensagem foi verificada pelo sistema de antiv�rus e
 acredita-se estar livre de perigo.


=
Instru��es para entrar na lista, sair da lista e usar a lista em
http://www.mat.puc-rio.br/~obmlistas/obm-l.html
=


Re: [obm-l]

2019-12-13 Por tôpico Bernardo Freitas Paulo da Costa
On Fri, Dec 13, 2019 at 2:05 AM Pedro Angelo  wrote:
>
> Fiz as contas (multiplicador de lagrange, parece muita conta mas é bem
> fazível) e é isso mesmo. Se eu não errei nada, fica
>
> k = 1 / raíz[ n (n-1) ]
>
> e a resposta é que o máximo possível para a soma dos cubos é:
>
> (1 - 2/n) / (1 - 1/n)^(1/2)
>
> que curiosamente é uma função crescente de n. Não está definida para
> n=1 (de fato, o problema {x=0, x^2=1} é impossível), vale zero para
> n=2 (trivial de verificar diretamente), e tende a 1 à medida que n
> cresce. Suponho que haja alguma lição interessante a ser aprendida
> disso? Ou é só um monte de conta e ponto final? rsrsrs

Bom, você pode imaginar algo sobre quão perto você consegue chegar dos
eixos coordenados no plano x_1 + x_2 + ... + x_n = 0.  E também
lembrar que "a maior parte da esfera está no equador", então acaba
ficando mais fácil conforme n aumenta.  Outra coisa legal de pensar é
comparar a norma 2 com a norma 3 (faça um desenho).  É, eu sei, não
tem módulo, mas acho que ainda pode dar uma ideia interessante.

(E sim, Álgebra Linear em alta dimensão é muito legal, mas pode ser
meio contra-intuitivo para os desenhos de dimensão 2 e 3 que a gente
faz no quadro)

Abraços,
-- 
Bernardo Freitas Paulo da Costa

-- 
Esta mensagem foi verificada pelo sistema de antiv�rus e
 acredita-se estar livre de perigo.


=
Instru��es para entrar na lista, sair da lista e usar a lista em
http://www.mat.puc-rio.br/~obmlistas/obm-l.html
=


Re: [obm-l]

2019-12-12 Por tôpico Bernardo Freitas Paulo da Costa
On Thu, Dec 12, 2019 at 6:49 PM Esdras Muniz  wrote:
>
> Isso aí pode ir para o infinito: tome k real positivo arbitrário. Daí tome:
> (-k)+(-k)+...+(-k)+(n-1)k=0
> (-k)^3+(-k)^3+...+(-k)^3+((n-1)k)^3=k^3((n-1)^3-(n-1)).
> Esse último fator vai pra o infinito com k.

A soma dos quadrados é um.  O máximo (e o mínimo) existem e são
finitos.  Acredito que a resposta certa segue sua ideia (aliás, não é
a primeira vez que este problema aparece aqui), apenas fixando k tal
que a soma dos quadrados seja um.  Mas poderia ser diferente, e não
parei para pensar.

>> Em seg., 9 de dez. de 2019 às 20:29, gilberto azevedo
>>  escreveu:
>> >
>> > Sabendo que :
>> > x_1 + ... + x_n = 0
>> > x_1 ² + ... + x_n ² = 1
>> > Qual o valor máximo de x_1 ³ + ... + x_n ³ ?

Abraços,
-- 
Bernardo Freitas Paulo da Costa

-- 
Esta mensagem foi verificada pelo sistema de antiv�rus e
 acredita-se estar livre de perigo.


=
Instru��es para entrar na lista, sair da lista e usar a lista em
http://www.mat.puc-rio.br/~obmlistas/obm-l.html
=


[obm-l] Re: [obm-l] Re: [obm-l] Re: [obm-l] Dúvida

2019-12-05 Por tôpico Bernardo Freitas Paulo da Costa
a circunferência de raio zero. E só para esse caso e ainda aceitarmos 
> que quando há só um foco ele tanto é direito quanto esquerdo. Grato pelos 
> comentários. Mas as dúvida persistem.
>
> Saudações,
> PJMS
>
>
>
> Em qua., 4 de dez. de 2019 às 19:59, Pedro Angelo  
> escreveu:
>>
>> Em matemática, geralmente é mais útil que as definições dos objetos
>> importantes não excluam os casos particulares. Um quadrado é um
>> retângulo? Se vc quiser que a definição de "retângulo" inclua somente
>> quadriláteros com ângulos retos que não sejam quadrados, vc tem que
>> explicitar a parte do "não sejam quadrados" na definição. A definição
>> mais simples, "retângulo é um quadrilátero cujos ângulos são todos
>> retos" (como o nome já diz!) inclui o quadrado como caso especial. Uma
>> coisa parecida ocorre com a elipse. Se vc quiser excluir o círculo, vc
>> teria que especificar na definição que vc quer focos distintos. A
>> definição mais simples, que cita os focos como sendo "dois pontos", ao
>> invés de "dois pontos distintos", inclui o círculo como caso especial.
>> E é útill que inclua mesmo. Por exemplo, se vc pensar o círculo como
>> sendo um tipo especial de elipse, vc pode enunciar o seguinte teorema:
>> "A imagem de uma elipse por uma transformação afim é outra elipse."
>> Mas se vc achar que um círculo não é uma elipse, então o teorema (da
>> forma que foi enunciado) não vale mais. A questão é que praticamente
>> qualquer propriedade interessante apresentada por "elipses
>> não-circulares" também será compartilhada pelos círculos. É raro em
>> matemática vc precisar de uma elipse que seja proibida de ser um
>> círculo. Nunca vi ninguém definir elipse de uma forma que exclua os
>> círculos.
>>
>> Sobre a suavidade: da forma que vc escreveu, eu diria que está um
>> pouco ruim. Por exemplo, a função
>> F(x,y)=x^2-y^2
>> é uma função suave (vc consegue calcular dF/dx e dF/dy, por exemplo).
>> Mas vc diria que a equação F(x,y)=0 é uma "cônica suave"? Repare que
>> essa equação descreve duas retas que se cruzam na origem. Outras
>> funções problemáticas são F(x,y)=x^2+y^2 e F(x,y)=0.
>>
>> Se F(x,y) é um polinômio de segundo grau em x e y, então F(x,y)=0 é
>> uma cônica, e eu diria que essa cônica é "suave" se nenhum dos pontos
>> dela (pontos (x,y) tais que F(x,y)=0) satisfaz ao mesmo tempo dF/dx=0
>> e dF/dy=0. O fato de pelo menos uma das derivadas parciais de F ser
>> não-nula garante que não encontraremos problemas como os do parágrafo
>> acima.
>>
>> abraços!
>>
>>
>> Le mer. 4 déc. 2019 à 19:10, Pedro José  a écrit :
>> >
>> > Boa noite!
>> > Estou dando uma repassada nas cônicas para auxiliar um filho de um amigo.
>> > Dúvidas quanto à cônicas.
>> > Alguns trabalhos até de mestrandos apontam a circunferência como sendo uma 
>> > elipse, um caso particular.
>> > Aprendera que o limite de uma elipse quando a distância entre os focos 
>> > tendesse para zero era uma circunferência, não obstante a circunferência 
>> > não é uma elipse.
>> > A elipse tem dois focos. O que não ocorre na circunferência.
>> > A elipse pode ser definida como o lugar geométrico do plano em que a razão 
>> > entre a distância de um ponto ao foco direito e a distância entre esse 
>> > ponto e uma reta (diretriz direita) é constante e menor que 1 e igual a 
>> > excentricidade da cônica.
>> > A circunferência não suporta tal definição.
>> > Vejo muitos autores chamarem cônicas suaves.Significa que se escrevermos 
>> > uma equação quadrática com F(x,y)=0 a função F(x,y) é suave?
>> >
>> > Grato!
>> >
>> > Saudações,
>> > PJMS.
>> >
>> > --
>> > Esta mensagem foi verificada pelo sistema de antivírus e
>> > acredita-se estar livre de perigo.
>>
>> --
>> Esta mensagem foi verificada pelo sistema de antivírus e
>>  acredita-se estar livre de perigo.
>>
>>
>> =
>> Instru�ões para entrar na lista, sair da lista e usar a lista em
>> http://www.mat.puc-rio.br/~obmlistas/obm-l.html
>> =
>
>
> --
> Esta mensagem foi verificada pelo sistema de antivírus e
> acredita-se estar livre de perigo.



-- 
Bernardo Freitas Paulo da Costa

-- 
Esta mensagem foi verificada pelo sistema de antiv�rus e
 acredita-se estar livre de perigo.


=
Instru��es para entrar na lista, sair da lista e usar a lista em
http://www.mat.puc-rio.br/~obmlistas/obm-l.html
=


[obm-l] Re: [obm-l] Re: [obm-l] polinômio redutível ?

2019-11-10 Por tôpico Bernardo Freitas Paulo da Costa
Aproveito para repassar o email do Luís, com as correções que ele
efetuou sobre meu rascunho e, mais importante, a motivação do
problema.

On Wed, Nov 6, 2019 at 8:42 PM Luís Lopes  wrote:
>
> Sauda,c~oes, oi Bernardo,
>
> Alguns comentários preliminares:
>
> 1) obrigado ao Bernardo pelo tempo e paciência. Não é a primeira vez.
>
> 2) x em p(x) representa o comprimento do lado  na resolução algébrica do
> problema "construir ΔABC dados  ou ".
>
> A construção geométrica é interessante e admite duas soluções (triângulos).
> Mas apareceram dois outros triângulos estranhos (o lado  é negativo) e
> então algebricamente um polinômio de grau 4 vai aparecer. Daí o surgimento
> e interesse em p(x).

Uma pergunta: se existe uma construção geométrica para duas raízes
(pelo menos no caso em que há solução), isso não implica que deve
haver uma fatoração construtível deste polinômio?  Não sou
especialmente versado em construtibilidade, mas como já haverá um
fator q(x) construtível de grau 2, a divisão p(x)/q(x) também será, e
o quociente tendo grau 2 também implica que as raízes são
construtíveis.  Claro que pode ser mais interessante fatorar antes de
construir as raízes, mas se for o único caminho para obter as 2
outras...

> 3) análise de p(x) mostra que não adianta procurar por mais de duas soluções,
> ou seja, o problema admite 0, 1 ou 2 soluções.
>
> 4) com dados construtíveis, as raízes de p(x) são construtíveis também, ou 
> seja,
> p(x) admite soluções com \sqrt{} somente.
>
> 5) p(x) é fatorável como produto de dois polinômios quadráticos.
>
> 6) a cúbica resolvente de p(x) possui sempre uma raiz racional.
>
> Fim dos comentários.
>
> No teste
>
> 3) h=4sqrt(3); m=(3/2)sqrt(3); s=13
> já sabemos que as raízes serão complexas pois h>2m. A fatoração anuncia-se
> complicada.
>
> p(x) = 9x^4 + 156x^3 + 160x^2 - 7384x + 20164 = (Ax^2 + Bx + C)(Dx^2 + Ex + F)
>
> só consegui fazendo A=3 e em seguida resolvendo o sistema que surge igualando
> os outros coeficientes. B,D=3,E são camaradas. C,F assustadores:
>
> B = 26 - sqrt(2 (511 + sqrt(317905)))
> C = 1/78 (6578 + 26 sqrt(317905) - sqrt(2) (511 + sqrt(317905))^(3/2) + 684 
> sqrt(2 (511 + sqrt(317905
> E = 26 + sqrt(2 (511 + sqrt(317905)))
> F = 1/78 (6578 + 26 sqrt(317905) + sqrt(2) (511 + sqrt(317905))^(3/2) - 684 
> sqrt(2 (511 + sqrt(317905
>
> E estes valores não são únicos. Se numericamente foi trabalhoso 
> simbolicamente seria
> ainda mais.

De fato!  Volto aqui a fazer propaganda do python: a biblioteca sympy
tem uma boa coleção de algoritmos para manipulações simbólicas (sendo
útil inclusive para gabaritar provas de Cálculo I ;-)), com código
aberto, e várias pérolas da matemática até "contemporânea", como um
algoritmo de cálculo de limites com garantia de terminação finita para
uma classe relativamente grandes de expressões.

> >Juntando tudo, temos (3x^2 + uE + uY)(3x^2 + E + Y/u) = seu polinômio, e 
> >temos ("de fato")
> >apenas uma incógnita.
> Aqui teve um typo: (3x^2 + uEx + uY)(3x^2 + Ex + Y/u)

Isso, obrigado!

> Daqui pra baixo me perdi.

Mas adivinhou o caminho ;-)

> >Aí eu pedi para o computador calcular as raízes E do polinômio (de
> >quarto grau) que fica determinado pela equação do termo x^2.
> Que polinômio é esse ?
> 3Y/u + uE^2 + 3uY com (u + 1)E = 4s e aparece um E^4. É isso ?

Isso, porque u = 4s/E - 1, e 1/u vai dar um termo com (4s - E) no
denominador, daí ao eliminar ambos o E^2 vai ter dois fatores com E a
mais.

> >Deu o seguinte:
>
> 2*s +/- 2*sqrt(2)*sqrt(-h**2 + m**2 + s**2 +/- sqrt(h**4 - 2*h**2*m**2
> - h**2*s**2 + m**4 - 2*m**2*s**2 + s**4))
>
> Se você chamar T = m^2 + s^2 - h^2, dá para ficar mais bonitinho:
>
> 2s +/- 2sqrt(2)*sqrt(T +/- sqrt(T^2 - 4 m^2 s^2))
> >
> São quatro valores. Posso pegar qq um ? Digamos
> E_1 = 2s + 2sqrt(2)*sqrt(T + sqrt(T^2 - 4 m^2 s^2)). Aí acho u_1 = 4s/E_1 - 1.
>
> E encontro p(x) = (3x^2 + u_1E_1x + u_1Y)(3x^2 + E_1x + Y/u_1).
> Seria isso ?

Acho que pode pegar o que você quiser sim, a fatoração deveria ser "a
mesma" (as raízes conjugadas vêm juntas, então ao conjugar o E_1 em
E_j deveria aparecer a raiz conjugada junto e, a menos de permutação,
seria igual).  Talvez precise de um pouco mais de formalização neste
argumento (quando justamente as raízes não forem conjugadas complexas)
mas acho que dá certo.

> Abraços,
> Luís

Grande abraço,
-- 
Bernardo Freitas Paulo da Costa

-- 
Esta mensagem foi verificada pelo sistema de antiv�rus e
 acredita-se estar livre de perigo.


=
Instru��es para entrar na lista, sair da lista e usar a lista em
http://www.mat.puc-rio.br/~obmlistas/obm-l.html
=


[obm-l] Re: [obm-l] polinômio redutível ?

2019-11-06 Por tôpico Bernardo Freitas Paulo da Costa
Oi Luís, e demais colegas da lista.

On Tue, Nov 5, 2019 at 4:43 PM Luís Lopes  wrote:
> Considere o polinômio
>
> p(x)[h,m,s] = 9x^4 + 12s x^3 + 2(8h^2 - 20m^2 - s^2)x^2 + 4s(4m^2 - s^2)x + 
> (4m^2 - s^2)^2 .
>
> Fiz alguns testes para ver se p(x) pode ter suas raízes construtíveis. Esse 
> polinômio aparece
> na construção do triângulo dados .
>
> [...]
>
> Só testei para h,m,s > 0 mas se não errei nessas contas parece que podemos 
> fatorar p(x)[h,m,s]  como
>
> 9x^4 + 12s x^3 + 2(8h^2 - 20m^2 - s^2)x^2 + 4s(4m^2 - s^2)x + (4m^2 - s^2)^2  
> = (Ax^2 + Bx + C)(Dx^2 + Ex + F) com os coeficientes A,B,... F construtíveis.
>
> Daria para calcular os coeficientes dos dois polinômios do segundo grau em 
> função de h,m,s ?

Se não errei contas (e se o meu computador também não errou...), acho que sim.

Tome A = D = 3 (para eliminar algumas das variáveis).
Depois, observe que o quadrado Y^2 no termo independente poderia ser
fatorado como CF = Y^2 = (uY)*(Y/u), introduzindo uma nova variável u
e substituindo C e F.
(se Y = 0, a equação só tem os termos de grau 2 e maiores, o que
também é fatorável, só que de outra forma; com Y != 0, C e F são
diferentes de zero, e u está justificada)
Além disso, a equação do termo x^3 dá 12s = 3B + 3E, o que permite
eliminar B, sobrando apenas E e u.

Agora, vem uma última coincidência: o termo Y = (4m^2 - s^2) também
aparece no termo de primeiro grau, o que permite fatorar Y na equação
entre os termos multiplicando x:
4sY = BF + CE = BY/u + uYE se torna 4s = B/u + uE.

Mas então B + E = 4s = B/u + uE <=> B(1 - 1/u) = E(u - 1).  Se u = 1,
veja mais abaixo, senão isso dá B(u-1)/u = E(u-1) <=> B = uE.
Juntando com B + E = 4s de volta, temos (u + 1)E = 4s, o que permite eliminar u.

Juntando tudo, temos
(3x^2 + uE + uY)(3x^2 + E + Y/u) = seu polinômio, e temos ("de fato")
apenas uma incógnita.

Aí eu pedi para o computador calcular as raízes E do polinômio (de
quarto grau) que fica determinado pela equação do termo x^2.
Deu o seguinte:

2*s +/- 2*sqrt(2)*sqrt(-h**2 + m**2 + s**2 +/- sqrt(h**4 - 2*h**2*m**2
- h**2*s**2 + m**4 - 2*m**2*s**2 + s**4))

Se você chamar T = m^2 + s^2 - h^2, dá para ficar mais bonitinho:

2s +/- 2sqrt(2)*sqrt(T +/- sqrt(T^2 - 4 m^2 s^2))

(Curiosamente, o polinômio em E também é irredutível segundo o meu
computador, mas ele consegue achar as raízes...)

Abraços,
-- 
Bernardo Freitas Paulo da Costa

-- 
Esta mensagem foi verificada pelo sistema de antiv�rus e
 acredita-se estar livre de perigo.


=
Instru��es para entrar na lista, sair da lista e usar a lista em
http://www.mat.puc-rio.br/~obmlistas/obm-l.html
=


[obm-l] Re: [obm-l] Somatórios

2019-07-21 Por tôpico Bernardo Freitas Paulo da Costa
On Sat, Jul 20, 2019 at 10:38 PM Eduardo Henrique  wrote:
> Pessoal, podem me indicar algum material que explique como funcionam os 
> somatórios? Gostaria de algum que explicasse em que casos podemos inverter 
> somatórios, quais as condições... tanto pra finitos quanto pra infinitos. 
> Pode ser apenas nomes de livros que tenham isso que eu corro atrás. Valeu!

Concrete Mathematics, Graham-Knuth-Patashnik. Provavelmente um livro
que vai te abrir a mente, além de te ensinar a nunca mais ter medo de
somatórios.

Abraços
-- 
Bernardo Freitas Paulo da Costa

-- 
Esta mensagem foi verificada pelo sistema de antiv�rus e
 acredita-se estar livre de perigo.


=
Instru��es para entrar na lista, sair da lista e usar a lista em
http://www.mat.puc-rio.br/~obmlistas/obm-l.html
=


[obm-l] Re: [obm-l] Re: [obm-l] Re: [obm-l] Re: [obm-l] teoria dos números curiosidade

2019-07-03 Por tôpico Bernardo Freitas Paulo da Costa
On Wed, Jul 3, 2019 at 8:34 PM Claudio Buffara
 wrote:
> Infinitas.
> Basta usar recursivamente a relação  1/n = 1/(n+1) + 1/(n(n+1)), que cada vez 
> você obtém uma representação mais longa.
> 1/2 = 1/3 + 1/6 = 1/3 + 1/7 + 1/42 = 1/3 + 1/7 + 1/43 + 1/1806 = ...

Mais difícil, talvez, seria calcular qual o menor número de termos
necessários para representar p/q :)  Será que isso é NP completo?

Abraços,
--
Bernardo Freitas Paulo da Costa

-- 
Esta mensagem foi verificada pelo sistema de antiv�rus e
 acredita-se estar livre de perigo.


=
Instru��es para entrar na lista, sair da lista e usar a lista em
http://www.mat.puc-rio.br/~obmlistas/obm-l.html
=


Re: [obm-l] desigualdades

2019-06-10 Por tôpico Bernardo Freitas Paulo da Costa
Outra ideia: seja 4 = n, e considere x_i/(x_i + y_i), onde y_i é um
"deslocamento" dos x; ou seja, x = [a,b,...,c,d], y = [b,...,c,d,a]
têm cada um n elementos.  O último exemplo do Ralph mostra que x/(x+y)
pode estar arbitrariamente próximo de [1,1, ..., 1, 0].  Daí, se
estivermos neste caso, basta notar que d/(d+a) (enfim, o último
dividido pela soma deste com o primeiro) será menor do que d/(d+c), já
que a >= b >= ... >= c.  Assim, c/(c+d) + d/(d+a) <= c/(c+d) + d/(c+d)
= 1. Ufa!  Assim, com n termos, temos que a soma vale, no máximo, n-1.
(Para a desigualdade estrita é fácil, basta ver que nenhum dos outros
termos dá 1.)

Para completar a demonstração no caso geral, separe o caso "geral",
onde a sequência tem mais de um mínimo local, e o caso monótono
decrescente (como a escolha do "primeiro elemento" não importa, já que
é circular, este sempre pode ser escolhido como o maior de todos).

Outro agradecimento ao Ralph por ter sugerido "maximizar" a expressão.

On Mon, Jun 10, 2019 at 9:06 PM Ralph Teixeira  wrote:
>
> Ah, errei sim! Poderia ser a≥b≥c≥d≤a, claro! :-(
>
> On Mon, Jun 10, 2019, 21:55 Ralph Teixeira  wrote:
>>
>> Uma ideia: cada uma das 4 frações é <1... Se você mostrar que duas delas são 
>> ≤ 1/2, acabou o problema.
>>
>> Então, se a≤b≤c então a/(a+b)≤a/(a+a)=1/2, e idem para b/(b+c). De fato, se 
>> houver 3 números consecutivos em ordem crescente na lista cíclica (a,b,c,d), 
>> este argumento mata o problema.
>>
>> Agora, para não ter 3 em ordem crescente no ciclo, você vai ter que ter 
>> a≤b≥c≤d≥a (ou exatamente o contrário disso tudo, que é análogo). Mas então 
>> a/(a+b)≤1/2 e c/(c+d)≤1/2 também!
>>
>> Abraço, Ralph.
>>
>> P.S.: meu raciocínio parece ter muita "folga", errei algo? Alguém sabe se o 
>> máximo daquela expressão está perto de 3 mesmo?
>>
>>
>>
>> On Mon, Jun 10, 2019 at 11:12 AM Carlos Monteiro 
>>  wrote:
>>>
>>> Prove que se a, b, c, d são reais positivos, então
>>> a/(a+b)  +  b/(b+c) +  c/(c+d)   +   d/(d+a)  <  3
>>>


-- 
Bernardo Freitas Paulo da Costa

-- 
Esta mensagem foi verificada pelo sistema de antiv�rus e
 acredita-se estar livre de perigo.


=
Instru��es para entrar na lista, sair da lista e usar a lista em
http://www.mat.puc-rio.br/~obmlistas/obm-l.html
=


Re: [obm-l] Probabilidade

2019-05-28 Por tôpico Bernardo Freitas Paulo da Costa
On Tue, May 28, 2019 at 10:34 AM matematica10complicada
 wrote:
>
> Olá amigos, o que acham desse problema?
>
> Qual seria a resposta?
>
> João e Maria tem dois filhos, e sabe-se que um dos filhos é um menino. Se a 
> probabilidade de um filho ser do sexo masculino é igual a 50%, é correto 
> afirmar que a probabilidade de o outro filho do casal ser um menino é igual a:

O que você acha?  Como você pensou?

Abraços,
-- 
Bernardo Freitas Paulo da Costa

-- 
Esta mensagem foi verificada pelo sistema de antiv�rus e
 acredita-se estar livre de perigo.


=
Instru��es para entrar na lista, sair da lista e usar a lista em
http://www.mat.puc-rio.br/~obmlistas/obm-l.html
=


[obm-l] Re: [obm-l] teoria dos números

2019-05-28 Por tôpico Bernardo Freitas Paulo da Costa
On Sun, May 26, 2019 at 8:50 AM Daniel Quevedo  wrote:
>
> Calcule a soma dos 3 últimos algarismos do número 2003^2002^2001.

Oi Daniel.  Você já ouviu falar de congruências?  E do "pequeno
teorema de Fermat"?

Abraços,
-- 
Bernardo Freitas Paulo da Costa

-- 
Esta mensagem foi verificada pelo sistema de antiv�rus e
 acredita-se estar livre de perigo.


=
Instru��es para entrar na lista, sair da lista e usar a lista em
http://www.mat.puc-rio.br/~obmlistas/obm-l.html
=


Re: [obm-l] Escalonamento "estranho"

2019-03-06 Por tôpico Bernardo Freitas Paulo da Costa
On Tue, Mar 5, 2019 at 4:43 PM Vanderlei Nemitz  wrote:
>
> Boa tarde!
> Uma questão bem antiga do IME pede para que o sistema linear homogêneo seja 
> discutido pelo Teorema de Rouché.
> (3 - k)x +2y + 2z = 0
>  x + (4 - k)y +   z = 0
>2x +4y + (1 + k)z = 0
>
> Os valores de k para os quais o determinante da matriz dos coeficientes é 
> nulo são k = 1, k = 2 e k = 3, ou seja, para esses valores o sistema é 
> indeterminado.
> Simples!
>
> Porém, se resolver pelo teorema de Rouché, para determinar a característica 
> da matriz, encontro apenas 2 valores de k para os quais a característica é 2. 
> O outro valor "se perde" no caminho.
> O mesmo acontece se escalonar o sistema.
>
> Porque isso ocorre? Pelo fato de multiplicarmos por "zero" em algum momento?

Em geral, é muito mais perigoso dividir por zero.  Mas pode ser...
Como você escalonou esta matriz?


Aliás, porque chamar de "caracterísitica" algo cujo nome oficial
(mesmo em português no Brasil) é chamado de "posto"?  Nunca ouvi isso
antes, é uma invenção da novlíngua, para confundir e evitar que os
alunos aprendam lendo a Wikipedia??

Abraços,
-- 
Bernardo Freitas Paulo da Costa

-- 
Esta mensagem foi verificada pelo sistema de antiv�rus e
 acredita-se estar livre de perigo.


=
Instru��es para entrar na lista, sair da lista e usar a lista em
http://www.mat.puc-rio.br/~obmlistas/obm-l.html
=


Re: [obm-l] multiplicadores de lagrnge

2019-03-06 Por tôpico Bernardo Freitas Paulo da Costa
On Sun, Mar 3, 2019 at 4:27 PM Israel Meireles Chrisostomo
 wrote:
>
> olá pessoal eu estava tentando encontrar um mínimo para a função 
> (a+b)z+(a+c)y+(b+c)x segundo a seguinte restrição xy+xz+yz+ab+bc+ac=2, pelo 
> método dos multiplicadores de lagrange eu encontrei 2 como ponto crítico, mas 
> calculando a hessiana aumentada não deu em nada, alguém ai poderia me ajudar, 
> por favor

a,b,c,x,y,z são positivos?

O que você tentou fazer mais claramente?  2 é um valor crítico (de
quem???)?  Quais são os valores de a,b,c,x,y,z que mostram isso?

Abraços,
-- 
Bernardo Freitas Paulo da Costa

-- 
Esta mensagem foi verificada pelo sistema de antiv�rus e
 acredita-se estar livre de perigo.


=
Instru��es para entrar na lista, sair da lista e usar a lista em
http://www.mat.puc-rio.br/~obmlistas/obm-l.html
=


Re: [obm-l] multiplicadores de lagrange

2019-03-06 Por tôpico Bernardo Freitas Paulo da Costa
On Thu, Feb 28, 2019 at 5:58 PM Israel Meireles Chrisostomo
 wrote:
>
> Sejam  f e g função de várias variáveis
> Se g é uma restrição, é verdade que a fórmula ∇ f=m∇g
> também vale para qualquer número de variáveis, ou só vale para 3 e 2 
> variáveis.

Se você interpretar as operações corretamente, isso vale inclusive
para  g:R^n -> R^k um conjunto de restrições, m será um vetor de
multiplicadores de Lagrange, e ∇g a Jacobiana. (Talvez você tenha que
transpor algumas coisas para as dimensões ficarem certas, mas só tem
um jeito que funciona para todos n,k)

Claro que eu estou assumindo que o resultado é "num ponto de ótimo
local, vale ∇f = m∇g" (mas isso não basta para caracterizar o ótimo,
tem condições de segunda ordem, da mesma forma que no caso de uma
variável, sem restrições)

Abraços,
-- 
Bernardo Freitas Paulo da Costa

-- 
Esta mensagem foi verificada pelo sistema de antiv�rus e
 acredita-se estar livre de perigo.


=
Instru��es para entrar na lista, sair da lista e usar a lista em
http://www.mat.puc-rio.br/~obmlistas/obm-l.html
=


Re: [obm-l] Matriz e determinante

2019-02-19 Por tôpico Bernardo Freitas Paulo da Costa
On Tue, Feb 19, 2019 at 7:45 AM Claudio Buffara
 wrote:
>
> Toda matriz tem um autovalor. De fato, uma matriz nxn tem n autovalores, que 
> podem não ser reais e nem todos distintos.

Tem um detalhe que não afeta a validade da sua demonstração, mas acho
importante ressaltar.  A definição de autovalores que dá n autovalores
para uma matriz nxn é "algébrica", e não vem necessariamente com n
autovetores associados (que são ditos "geométricos").  Isso acontece
com matrizes nilpotentes, por exemplo, e de forma mais geral em blocos
de Jordan (como o artigo mostra).  O importante (que vem justamente de
Jordan) é que para cada autovalor k (SEM multiplicidade, portanto
podendo ser menos do que n) sempre tem pelo menos um autovetor X
correspondente, e daí você pode usar o seu raciocínio para mostrar que
Re(k) = 1/2.  Daí você volta para a álgebra, e vê que as
multiplicidades (algébricas) dos autovalores conjugados são iguais.
(Talvez até seja possível mostrar que se M é real, as multiplicidades
geométricas de autovalores conjugados também são iguais, mas neste
caso não precisa)

> Dá uma olhada nesse artigo aqui: 
> https://www.maa.org/sites/default/files/pdf/awards/Axler-Ford-1996.pdf

Muito bom!!  Vou usar nas minhas aulas de Álgebra Linear.  Uma coisa,
entretanto: não vi a demonstração (nem o enunciado) do fato geométrico
"há pelo menos um autovetor de verdade para cada autovalor" (e não
apenas autovetores generalizados).  Não é difícil com tudo o que já
tem no artigo: por exemplo, (T - lambda * I)^k v = 0 mas não com k-1
implica que w = (T - lambda I)^{k-1} v é um autovetor "normal" de T.

> []s,
> Claudio.
>
>
> On Tue, Feb 19, 2019 at 9:23 AM Rodrigo Ângelo  wrote:
>>
>> Oi, Claudio
>>
>> Nesse caso, como a gente sabe que A tem um auto valor k?
>>
>> Atenciosamente,
>> Rodrigo de Castro Ângelo
>>
>>
>> Em seg, 18 de fev de 2019 às 22:25, Claudio Buffara 
>>  escreveu:
>>>
>>> Dada uma matriz qualquer M, vou chamar de M* a conjugada transposta de M 
>>> (se M for real, M* = transposta de M).
>>> Dado um número complexo z, chamarei de z* o conjugado de z.
>>> E identificarei números complexos com matrizes 1x1.
>>>
>>> Seja k um autovalor de A.
>>> Então existe uma matriz coluna nx1 não nula X tal que AX = kX ==> X*A* = 
>>> k*X*
>>> X*AX = X*(kX) = kX*X
>>> X*A*X = (k*X*)X = k*X*X
>>>
>>> Somando estas duas equações, obtemos:
>>> X*AX + X*A*X = (k+k*)X*X ==>
>>> X*(A + A*)X = 2Re(k)X*X ==>
>>> X*IX = 2Re(k)X*X ==>
>>> X*X = 2Re(k)X*X ==>
>>> (1 - 2Re(k))X*X = 0.
>>>
>>> Como X <> 0, X*X > 0 ==> Re(k) = 1/2.
>>>
>>> Ou seja, todos os autovalores de A têm parte real = 1/2.
>>> Como A é real, o polinômio característico de A tem coeficientes reais ==>
>>> os autovalores de A ou são reais (e iguais a 1/2) ou então podem ser 
>>> particionados em pares da forma 1/2 + ib, 1/2 - ib (b real), cujo produto é 
>>> 1/4 + b^2 > 0 ==>
>>> det(A) = produto dos autovalores de A > 0.
>>>
>>> []s,
>>> Claudio.
>>>
>>>
>>>
>>>
>>> On Mon, Feb 18, 2019 at 9:50 PM Vanderlei Nemitz  
>>> wrote:
>>>>
>>>> Pessoal, estou pensando na seguinte questão, consegui alguns resultados, 
>>>> mas nada concreto. Alguém com uma ideia que possa resolver?
>>>>
>>>> Seja A uma matriz real n x n tal que A + A^t = I.
>>>> Prove que detA > 0.
>>>>
>>>> A^t é a transposta de A.
>>>>
>>>> Muito obrigado!
>>>>
>>>> Vanderlei


-- 
Bernardo Freitas Paulo da Costa

-- 
Esta mensagem foi verificada pelo sistema de antiv�rus e
 acredita-se estar livre de perigo.


=
Instru��es para entrar na lista, sair da lista e usar a lista em
http://www.mat.puc-rio.br/~obmlistas/obm-l.html
=


Re: [obm-l] Re: |P(z)| > |Q(z)| para uma infinidade de z's

2019-02-11 Por tôpico Bernardo Freitas Paulo da Costa
On Mon, Feb 11, 2019 at 11:11 AM Artur Steiner
 wrote:
> OK.

Eu também fiz assim, à primeira vista.  Na "força bruta", analisando
os 3 casos (deg P > deg Q, e os dois casos deg P < deg Q).  É um pouco
mais satisfatório do que usar os canhões de análise complexa, porque a
gente fica com a impressão que dá para usar em casos mais gerais.  Mas
acaba sendo uma demonstração de uma natureza bem diferente.

> Um problema mais interessante é provar que, se f e g são funções inteiras 
> tais que |f(z)| > |g(z)| ocorra para um número finito de complexos, então, 
> para todo z, f(z) = k g(z), sendo k uma constante com |k| <= 1.

E daí f nunca é maior do que g, logo finito = 0 ;-)

Na verdade, como f/g é uma função meromorfa, sua imagem é ou constante
ou "quase total" (a menos de 2 pontos, como é o caso da exponencial,
que não pega 0 nem infinito), pelo teorema de Picard.  Isso é (muito)
mais forte do que o seu resultado.  Mesmo Casorati-Weierstrass mostra
que a imagem de f/g é densa na esfera de Riemann, o que já é também
mais forte do que o seu resultado.

> Isso também poderia ser aplicado no caso dos polinômios. Se |Pz)| > |Q(z)| 
> para um número finito de complexos, concluímos que P é Q têm o mesmo grau.

Aqui é um pouco diferente do que eu fiz acima, porque P/Q não tem uma
singularidade essencial no infinito.  Mas daí eu faria de um outro
modo (se é para usar complexos):  Considere a equação P/Q = a, com a
um número complexo qualquer.  Pelo Teorema Fundamental da Álgebra, P -
a*Q (que não é nulo porque os graus são diferentes) tem uma raiz.
Daí, variando a (no complementar do disco unitário) você encontra uma
quantidade não enumerável (vezes o grau de P - a*Q) de pontos z onde
P/Q = a, e portanto |P| > |Q|


Como você mesmo falou, o importante é o resultado de funções inteiras.
A hipótese que os graus de P e Q são distintos é "quase para
confundir", porque parece que tem algo especial nisso, e na verdade
não: basta garantir que P/Q não seja constante.

Abraços,
-- 
Bernardo Freitas Paulo da Costa

-- 
Esta mensagem foi verificada pelo sistema de antiv�rus e
 acredita-se estar livre de perigo.


=
Instru��es para entrar na lista, sair da lista e usar a lista em
http://www.mat.puc-rio.br/~obmlistas/obm-l.html
=


Re: [obm-l] Tipo de primos

2019-02-10 Por tôpico Bernardo Freitas Paulo da Costa
On Mon, Feb 11, 2019 at 1:28 AM Luiz Kv  wrote:
>
> Oi pessoal, tudo bom ? Eu tava mexendo aqui pensando sobre numeros primos, e 
> percebi que tem vários primos que são obtidos fazendo a multiplicação de um 
> numero par por ele +2 + ele mais 1, tipo:
> 2*4 + 3 = 11, primo
> 20*22 + 21 = 461, primo
> 48*50 + 48 = 2449, primo
>
> se proceder assim com todos os pares até 64 (64*66 + 65 = 4289 que eh primo 
> tbm) dá 18 que são primos, ou seja, mais da metade.
>
> algm sabe me dizer se isso é algo especial de números primos ou eu tô 
> viajando, ou se é algo óbvio e eu não percebi, e se for especial qual o nome 
> desses tipos?

Bom, vou arriscar.  Estes números são da forma m(m+2) + (m+1) = m^2 +
3m + 1, com m par isso dá 4n^2 + 6n + 1.  Não acho que tem nada muito
surpreendente de haver vários deles que são primos quando m (ou n) é
pequeno.  Por exemplo, o polinômio do segundo grau n^2 + n + 41 gera
um monte de números primos, veja
https://math.stackexchange.com/questions/289338/is-the-notorious-n2-n-41-prime-generator-the-last-of-its-type.
O seu caso é ligeiramente diferente (você não pede que sejam
consecutivos), mas não sei se muda muito.

Abraços,
-- 
Bernardo Freitas Paulo da Costa

-- 
Esta mensagem foi verificada pelo sistema de antiv�rus e
 acredita-se estar livre de perigo.


=
Instru��es para entrar na lista, sair da lista e usar a lista em
http://www.mat.puc-rio.br/~obmlistas/obm-l.html
=


[obm-l] Re: [obm-l] Re: [obm-l] Re: [obm-l] [Problema] Achar o mínimo do valor absoluto de uma soma complexa

2018-11-12 Por tôpico Bernardo Freitas Paulo da Costa
Oi,

acho que você interpretou o enunciado de forma a "evitar os
complexos".  O problema original fala de "achar um ponto dentro do
círculo", então talvez não sejam apenas os pontos na circunferência
(como parece que a sua solução faz, ao ordenar todos pelos ângulos
centrais), mas qualquer ponto da forma r*cis(theta).  E daí talvez
tenha mais a ver com complexos...

On Mon, Nov 5, 2018 at 4:51 PM Pedro José  wrote:
>
> Boa tarde!
> Se entendi o que você quer, não entendi qual a relação com o mínimo de uma 
> soma complexa?
> Para resolver o problema que você propõe, entendi:
> (i) a excursão como a geração de um setor circular, a partir de um ponto 
> inicial, essa incursão tem dois sentidos, trigonométrico ou horário.
> (ii) Englobar um ponto significa que o ponto pertença ao setor circular, tem 
> que saber se incluem-se os pontos de borda ou não, como não há restrição vou 
> considerar que sim.
> (iii) Estou supondo que seu universo é plano.
> Minha sugestão é defina o conjunto de pontos em coordenadas polares.
> Defina a variável excursão, e dê a ela um sinal para definir o sentido,
> Faça um programa.
> Definir "arrays" dos pontos (caso não estejam em coordenadas polares, tem que 
> fazer uma sub-rotina para transformar as coordenadas em polares) Mod(I) e 
> Teta(I)
> Defina um array de contagens
> Definir uma rotina para contar o número de pontos. N
> Defina uma sub-rotina Achapontos para determinar o índice Imax, cujo 
> Engloba(I) seja máximo.
> ! Comentário: Atentar que podem retornar mais do que um índice. Portanto 
> deve-se definir um array Pontonotável e uma variável de contagem Nmax. E.g., 
> se tiverem três pontos que englobem o número máximo de pontos, deve retornar: 
> Array contagem, com os valores dos índices dos pontos que têm o máximo de 
> Engloba, nas três primeiras posições e o Valor Nmax=3.
> Aplique a sub-rotina de contagem no array de pontos e retorne com N.
> Entre com o valor de excursão
> Faça de I=1 a N
> Tetamax= max (teta(I);teta(i)+excursão)
> Tetamin=min(teta(I);teta(i)+excursão)
> Engloba(I)=0
> Faça de J=1 a N
> Se (teta(J)<=tetamax e teta(J)>=tetamin e mod(J)<=mod(I).
> ! comentário: A relação engloba será reflexiva. Todo ponto engloba si 
> próprio.Caso não se aceite a borda é só tirar os iguais da lógica acima.
> Engloba(I) = Engloba(I)+1
> Fim SE;
> Fim Faça
> Fim faça
> Aplica Sub-rotina acha pontos.
> Salva temos o máximo de pontos englobados para uma excurção de [excursão] 
> para [Nmax] pontos englobando [engloba(Nmax)]
> São eles:
> Faça de I=1 até N
> Pontonotável(I)
> Fim faça.
> !se tiver interesse salva todo array Engloba.
> FIM.
>
> Porém para qualquer setor existir um ponto que seja sempre o que englobe mais 
> pontos, creio que vá depender da nuvem, e.g.
>
> P1= (10,40)
> P2= ( 6,42)
> P3= (9,90)
> P4= (8,100)
> P5= (7,107)
> P6= (7,5; 108)
> Teremos para uma excursão de + 5 graus:
> Engloba (1) = 1;Engloba (2) = 0; Engloba (3) = 0; Engloba (4) = 0 , Engloba 
> (5) = 0 e Engloba (6) = 0
> P1 é o que engloba mais pontos.
> Para uma excursão de +10 graus:
> Engloba (1) = 1;Engloba (2) = 0; Engloba (3) = 1; Engloba (4) = 2 e Engloba 
> (5) = 0 e Engloba (6) = 0
> P3 é o que engloba mais pontos.
>
> Espero ter compreendido o proposto e ajudado.
> Mas o que tem haver com soma de complexos, módulo mínimo???
>
> Saudações,
> PJMS
>
> Em sáb, 3 de nov de 2018 às 22:31, Bruno Visnadi 
>  escreveu:
>>
>> Não entendi a pergunta - o que é uma excursão?
>>
>> Em sáb, 3 de nov de 2018 às 22:18, Jardiel Cunha  
>> escreveu:
>>>
>>> Olá!
>>>
>>>
>>> Estou trabalhando em um projeto e um problema está me tirando o sono há 
>>> algum tempo. Meu trabalho é na área de engenharia de microondas. A solução 
>>> que eu encontrei até agora, acha soluções mas não satisfatórias... Não 
>>> precisam fazer o problema, queria apenas uma luz em que caminho seguir.
>>>
>>>
>>> [Problema] Dados N pontos em um círculo, estou querendo achar um ponto 
>>> dentro do círculo tal que: para qualquer valor de excursão em graus, eu 
>>> garanta que não existe outro ponto que englobe mais pontos no círculo do 
>>> que ele.
>>>
>>>
>>> Por exemplo: se eu der uma excursão de 80 graus... então eu quero um ponto 
>>> tal que englobe o maior número possível desses N pontos estando ele no 
>>> centro de um arco de 80 graus.
>>>
>>>
>>> Mais um exemplo: tenho 10 pontos. Queria um ponto x tal que ele será o 
>>> centro de todos os arcos com o maior número possível de pontos.
>>>
>>>

Re: [obm-l] Probabilidade

2018-11-12 Por tôpico Bernardo Freitas Paulo da Costa
On Wed, Nov 7, 2018 at 3:28 PM Paulo Rodrigues  wrote:
>
> Muito obrigado pelos avanços.
>
> Se der pra calcular o valor exato melhor, mas se desse pra estimar essa 
> probabilidade, eu ficaria satisfeito. Depois explico o contexto prático do 
> problema.

Se for só "estimar", eu sugiro dar uma olhada em combinatória
analítica: http://algo.inria.fr/flajolet/Publications/AnaCombi/anacombi.html

Abraços,
-- 
Bernardo Freitas Paulo da Costa

-- 
Esta mensagem foi verificada pelo sistema de antiv�rus e
 acredita-se estar livre de perigo.


=
Instru��es para entrar na lista, sair da lista e usar a lista em
http://www.mat.puc-rio.br/~obmlistas/obm-l.html
=


[obm-l] Re: [obm-l] Re: [obm-l] Re: [obm-l] Re: [obm-l] Dúvida conceitual (equações)

2018-10-15 Por tôpico Bernardo Freitas Paulo da Costa
On Mon, Oct 15, 2018 at 8:07 AM Claudio Buffara
 wrote:
>
> Derivando e igualando a zero o lado esquerdo da sua equação, ficamos com:
> -2*cos(x)*sen(x) + sen(x) = 0 ==>
> sen(x) = 0  ou  cos(x) = 1/2 ==>
> x = 0 ou x = pi ou x = 2pi
> ou x = pi/3 ou x = 5pi/3.
>
> Assim, uma definição que me parece adequado para equações em geral (e não 
> necessariamente polinomiais) da forma f(x) = 0 é que uma raiz de 
> multiplicidade n é raiz de f, f’, ... , f^(n-1) mas não é raiz de f^(n).
>
> Naturalmente, se f não tiver todas as derivadas, precisaremos achar uma 
> definição diferente. Mas talvez, neste caso, nem faça sentido falar em 
> multiplicidade de uma raiz.

Essa definição funciona relativamente bem se f é analítica, porque o
comportamento local é determinado por inteiros.  Se f for apenas
diferenciável, talvez seja complicado dizer algo, como o exemplo
clássico de exp(-1/x^2).  A raiz tem multiplicidade infinita?

Enfim, existem, como você falou, boas razões para incorporar
multiplicidade (por exemplo estabilidade numérica), mas isso em geral
só faz sentido no mundo analítico, onde a noção de "grau" é dada pelas
derivadas.  Acho que mesmo no mundo C-infinito já pode haver
problemas, mas não sou especialista (nessas :D) patologias.  A questão
original, incluindo multiplicidades, pode ser resolvida simplesmente
usando as relações de Girard, que dependem de forma simples da
equação.

Vou tentar dar um exemplo que ilustra meu ponto de vista:  qual o
produto das raízes da equação x^2 - 4x + c?  "Qualquer um" dirá "c".
Mas, naturalmente, se c = 4, a única solução é x=2, e portanto (sem
usar multiplicidades) este produto seria apenas 2.  E daí a fórmula
fica muito mais complicada, com um caso especial, e descontínua.  A
grande sacada do Girard foi, justamente, propor incorporar as
multiplicidades, para simplificar as fórmulas (além, é claro, de
incluir também as soluções negativas, antes consideradas como
"absurdas" - este foi, provavelmente, o maior motivo de as pessoas
considerarem raízes negativas como algo que fazia sentido, e portanto
os números negativos também).  Mas isso não quer dizer que a equação
x^2 - 4x + 4 tenha duas soluções.  É apenas uma forma mais conveniente
de interpretar as raízes quando se pensam nas relações de Girard (e
várias outras fórmulas).  Neste sentido, acho que este tipo de questão
mais atrapalha (porque "era só para usar a fórmula") - a menos que,
justamente, se discuta *porque* falamos de multiplicidade: para que as
fórmulas fiquem mais simples (e você pode incluir "bonitas" também,
por minha conta).  Nada mais.  E esta "simplificação" do entendimento
através da simplificação das fórmulas não se justifica sempre: este
mesmo debate sobre multiplicidades leva a considerar objetos no
infinito (para que todas as retas se intersectem sempre em um ponto),
complexos (para x^2 + 1 = 0 ter raiz), etc.  Muitas vezes, é útil ter
esse entendimento unificado, onde tudo "só depende do grau".  Mas será
mesmo que se eu perguntar para você "em quantos pontos a reta x=3
corta a parábola y=x^2?" você vai dizer "2, é óbvio"?

Abraços,
-- 
Bernardo Freitas Paulo da Costa

-- 
Esta mensagem foi verificada pelo sistema de antiv�rus e
 acredita-se estar livre de perigo.


=
Instru��es para entrar na lista, sair da lista e usar a lista em
http://www.mat.puc-rio.br/~obmlistas/obm-l.html
=


[obm-l] Re: [obm-l] Aritmética

2018-09-05 Por tôpico Bernardo Freitas Paulo da Costa
On Wed, Sep 5, 2018 at 7:17 PM Israel Meireles Chrisostomo
 wrote:
> Olá pessoal, como posso provar que se a,b,c,d, são positivos e se a>b, c>d 
> então ac>bd

Oi Israel, Pedro, Luciano, e demais colegas da lista,

quais são os resultados que você pode usar para demonstrar isso?
Positivos quer dizer reais, eu imagino, mas dependendo de como você
define / constrói os reais, a forma de responder (e entender) esta
questão é diferente.  Por exemplo, todas as manipulações "algébricas"
(do tipo "a > b => a/b > 1") já podem pedir uma demonstração das
mesmas... Tudo depende do que você assume / admite como conhecido.

Abraços,
-- 
Bernardo Freitas Paulo da Costa

-- 
Esta mensagem foi verificada pelo sistema de antiv�rus e
 acredita-se estar livre de perigo.


=
Instru��es para entrar na lista, sair da lista e usar a lista em
http://www.mat.puc-rio.br/~obmlistas/obm-l.html
=


[obm-l] Re: [obm-l] Potenciação de complexos

2018-08-31 Por tôpico Bernardo Freitas Paulo da Costa
On Thu, Aug 30, 2018 at 9:55 PM Israel Meireles Chrisostomo
 wrote:
>
> Olá pessoal, eu gostaria de saber se a seguinte manipulação com complexos é 
> verdadeira:
> (m+ni)^{xy}=((m+ni)^x)^y
> Onde m,n,x,y são reais e i a unidade imaginária.

Nem precisa de complexos para ser falso:

Seja m = -1, x = 2, y = 0.5.  A primeira conta dá (-1)^(2 * 0.5) =
(-1)^1 = -1, enquanto a segunda dá ((-1)^2)^0.5 = 1^(0.5) = 1.

Abraços,
-- 
Bernardo

-- 
Esta mensagem foi verificada pelo sistema de antiv�rus e
 acredita-se estar livre de perigo.


=
Instru��es para entrar na lista, sair da lista e usar a lista em
http://www.mat.puc-rio.br/~obmlistas/obm-l.html
=


[obm-l] Re: [obm-l] Re: [obm-l] Re: [obm-l] Re: [obm-l] Re: [obm-l] Re: [obm-l] Equações do 2 grau

2018-08-23 Por tôpico Bernardo Freitas Paulo da Costa
On Thu, Aug 23, 2018 at 10:29 AM Artur Steiner
 wrote:
> É, inverter a ordem dos coeficientes foi genial.
E dá para fazer ao contrário também: a^2 + ab + ac < 0 quer dizer que
f(a) < 0, com f(x) = x^2 + bx + ac.  Isso novamente implica que a
equação f(x) = 0 tem duas soluções, logo o discriminante é < 0, o que
dá b^2 > 4ac.
-- 
Bernardo Freitas Paulo da Costa

-- 
Esta mensagem foi verificada pelo sistema de antiv�rus e
 acredita-se estar livre de perigo.


=
Instru��es para entrar na lista, sair da lista e usar a lista em
http://www.mat.puc-rio.br/~obmlistas/obm-l.html
=


[obm-l] Re: [obm-l] Mostrar que f é identicamente nulo.

2018-08-14 Por tôpico Bernardo Freitas Paulo da Costa
2018-08-14 17:03 GMT-03:00 Artur Steiner :
> Suponhamos que f: [0, 1] ---> R seja contínua e que, para todo n = 0, 1,
> 2.., Integral [0, 1] f(x) x^n dx = 0. Mostre que f é identicamente nula.
>
> Isso parece um tanto intuitivo, mas será que há uma prova imediata ou quase?

Não sei exatamente o que você espera.  A demonstração "abstrata" é que
a sua condição diz que f é ortogonal aos polinômios no intervalo [0,1]
(usando a métrica  = int f(x)g(x) dx).  Mas daí f também é
ortogonal ao limite uniforme de polinômios: basta tomar o limite P_n
-> g e multiplicar por f; sendo contínua, a convergência f*P_n -> f*g
também é uniforme, e portanto 0 = int(f*P_n) -> int(f*g).  Como os
polinômios são densos nas funções contínuas, isso mostra que f é
ortogonal a qualquer função g.  Em particular, ela é ortogonal a ela
mesma, e portanto f = 0.

Tendo feito isso, dá para simplificar um pouco a demonstração.  Por
exemplo, basta aproximar f = lim P_n (limite uniforme de polinômios) e
calcular int f(x)^2 = int f(x)*[f(x) - P(x) + P(x)] = int f(x)*[f(x) -
P(x)] + 0.  A integral restante é menor (em módulo) do que eps *
max(|f|) (já que aproximamos f por P_n com erro uniforme menor do que
eps).  Tomando n -> infinito, temos que int f(x)^2 -> 0 e daí f deve
ser identicamente nula.

Tive uma idéia agora de como fazer sem usar que dá para aproximar f
por polinômios, mas também aproximando.  Acho inevitável aproximar.
Tome uma aproximação linear por partes de f.  Como f é uniformemente
contínua, basta escolher pontos espaçados de delta para garantir um
erro uniforme < eps.  Agora, mostre que uma função g linear por partes
satisfazendo int g(x) x^n = 0 para todo n implica que g == 0.  Daí
conclua como antes.  Dá inclusive para fazer com aproximações
*constantes* por partes, o que possivelmente simplifica a demonstração
int g(x)x^n =0 => g == 0.

Abraços,
-- 
Bernardo Freitas Paulo da Costa

-- 
Esta mensagem foi verificada pelo sistema de antiv�rus e
 acredita-se estar livre de perigo.


=
Instru��es para entrar na lista, sair da lista e usar a lista em
http://www.mat.puc-rio.br/~obmlistas/obm-l.html
=


Re: [obm-l] Ajuda em desigualdade

2018-07-06 Por tôpico Bernardo Freitas Paulo da Costa
2018-07-02 8:38 GMT-03:00 marcone augusto araújo borges
:
> Sejam x, y e z números positivos tais que x+y+z = 9, determine o valor
> mínimo de P =(x^3 + y^3)\(xy+9) + (x^3 +z^3)\(xz+9) + (y^3 + z^3)\(yz+9)

Bom, acredito que resolvi.  Com uma ajuda do computador para
- fazer uns gráficos (1D)
- calcular derivadas simbólicas
- calcular uns valores numéricos

Vou assumir que é a soma simétrica de (x^3 + y^3) / (xy + 9) (note a
posição da barra, eu pensei durante algum tempo que era o inverso
disso... A\b é uma notação comum para inversa à esquerda... enfim :D).
Chame essa soma de S.

Primeiro, pense em min (x^3 + y^3) / (xy + 9) com a condição que x+y =
a.  Esse é um problema mais fácil, a simetria dá que a solução ou é
simétrica x=y, ou é "extrema" x=0, y = a (ou simetricamente x=a, y=0).
Calculando, x=y=a/2 dá  g(a) = 2*(a/2)^3 / ((a/2)^2 + 9) = a^3/4 /
(a^2/4 + 3^2) = a^3 / (a^2 + 6^2)  enquanto que as outras (extremas)
dão a^3/9.  Como 36 > 9, a primeira é sempre menor, independente de a.

Agora, repita para os outros 2 pares, impondo as condições x+y=a,
y+z=b, z+x=c e você terá a soma simétrica S2 = g(a) + g(b) + g(c), com
a condição que a+b+c = 18.  Como isso é um monte de mínimos separados,
esta nova soma S2 é uma estimativa por baixo de S.  Só que g é convexa
até x = 6*sqrt(3) (eu pedi ajuda pro maxima pra calcular a segunda
derivada, mas dá para usar o wolfram alpha), e portanto:
1) ou a soma simétrica é minimizada em a=b=c=6 (convexidade => mínimo
no ponto de simetria), ou
2) algum deles é maior do que 6*sqrt(3).

Mas o caso 2 é impossível, porque teríamos x+y = a = 6*sqrt(3) > 10 >
9 = x+y+z, absurdo porque z > 0.

O caso 1 implica x=y=z=3, e isso demonstra que (3,3,3) é de fato mínimo global.




Para os que querem todos os detalhes, segue a demonstração de que S2 <= S

S2 = min_{a,b,c} g(a) + g(b) + g(c) s.a. a+b+c = 18
S = min_{x,y,z} P(x,y) + P(y,z) + P(z,x) s.a. x+y+z = 9

Em S, introduza as variáveis a,b,c como indicado

S = min_{a,b,c,x,y,z} P(x,y) + P(y,z) + P(z,x) s.a. x+y+z = 9, x+y=a,
y+z=b, z+x=c, a+b+c = 18

E adicione novas variáveis "cópia" X,Y,Z.

S = min_{a,b,c,x,y,z,X,Y,Z} P(x,Y) + P(y,Z) + P(z,X)
s.a. x+y+z = 9, x+Y = a, y+Z = b, z+X = c, a+b+c = 18, x=X, y=Y, z=Z

Agora, "esqueça" as restrições "que me complicam": x+y+z = 9, x=X,
y=Y, z=Z.  Isso reduz o valor do mínimo (pois temos mais
flexibilidade), e "desacopla" o problema em um "mestre" e um
"subordenado":

S_modif = min_{a,b,c} [min_{x,y,z,X,Y,Z} P(x,Y) + P(y,Z) + P(x,Z) s.a.
x+Y=a, y+Z=b, z+X=c] s.a. a+b+c = 18

Mas o subordenado está desacoplado também: é igual a [min_{x,Y} P(x,Y)
s.a. x+Y = a] + [idem y,Z] + [idem z,X] = g(a) + g(b) + g(c) pela
nossa definição de g.

Isso prova que S_modif = S2, e porque "retiramos restrições", S2 <= S.

O curioso é que, mesmo introduzindo 6 novas variáveis, com 4+4 novas
restrições (de compatibilidade), e jogando 5 restrições no lixo, ainda
temos S2 = S.


Abraços,
-- 
Bernardo Freitas Paulo da Costa

-- 
Esta mensagem foi verificada pelo sistema de antiv�rus e
 acredita-se estar livre de perigo.


=
Instru��es para entrar na lista, sair da lista e usar a lista em
http://www.mat.puc-rio.br/~obmlistas/obm-l.html
=


[obm-l] Re: [obm-l] Equação 4 grau

2018-06-26 Por tôpico Bernardo Freitas Paulo da Costa
2018-06-26 15:09 GMT-03:00 Daniel Quevedo :
> As raizes reais da equação x^4 -4x=1 pertencem ao intervalo:

http://www.wolframalpha.com/input/?i=x%5E4+-4x%3D1

Dá uma solução em -0.24904... e outra em 1.6633...
Não tem um problema com o enunciado??

> A) (1,11)
> B) (2, 12)
> C) (3, 13)
> D) (4, 14)
> E) ( 5, 15)
>
> R: c

Abraços,
-- 
Bernardo Freitas Paulo da Costa

-- 
Esta mensagem foi verificada pelo sistema de antiv�rus e
 acredita-se estar livre de perigo.


=
Instru��es para entrar na lista, sair da lista e usar a lista em
http://www.mat.puc-rio.br/~obmlistas/obm-l.html
=


[obm-l] Re: [obm-l] Re: [obm-l] Teoria dos números

2018-06-02 Por tôpico Bernardo Freitas Paulo da Costa
2018-06-02 15:14 GMT-03:00 Claudio Buffara :
> Mudando a notação, eu pus N = X e R = X5.
>
> Então: R^5*N^5 + R*N = N^6 + R^6.

Essa mudança de notação é o pulo do gato!  Daqui, um pouco de
tentativa e erro faz a seguinte dedução:

R^5 N^5 - R^6 = N^6 - RN
R^5(N^5 - R) = N(N^5 - R)
(N - R^5)(N^5 - R) = 0

Então ou N^5 = R, ou N = R^5.
A primeira dá R = 0 ou 1 (únicas potências quintas entre 0 e 4), e com
isso N = 0 ou 1
A segunda dá R = 0,1,2,3,4, e N = respectivas potências 5as.  Que são
justamente as soluções que você encontrou!

> Com uma planilha, eu achei apenas 5 soluções:
> 0, 1, 32, 243, 1024.

Abraços,
-- 
Bernardo Freitas Paulo da Costa

-- 
Esta mensagem foi verificada pelo sistema de antiv�rus e
 acredita-se estar livre de perigo.


=
Instru��es para entrar na lista, sair da lista e usar a lista em
http://www.mat.puc-rio.br/~obmlistas/obm-l.html
=


[obm-l] Re: [obm-l] Re: [obm-l] Re: [obm-l] Esta equação trigonométrica tem raízes não reais?

2018-05-24 Por tôpico Bernardo Freitas Paulo da Costa
2018-05-24 13:29 GMT-03:00 Artur Steiner :
> Nâo tem mesmo nâo. Outra forma de ver isto é com a identidade sen(z) +
> cos(z) = raiz(2) sen(z + pi/4), Isto nos leva a
>
> sen(z + pi/4) = raiz(2)/2, que é um real em [-1, 1]. Logo, z + pi/4, e
> portanto z, são reais.
>
> Se sen(z) é um real em [-1, 1], então z é real. Condição similar vale para o
> cosseno.

Então dou mais uma ;-)

escrevendo sen(z) e cos(z) em função de w = exp(iz) e 1/w = exp(-iz),
temos uma equação quadrática em w.  Os coeficientes são complexos, mas
a fórmula funciona igual, e portanto temos duas soluções para w.
Substituindo o que já sabemos (z = 0, que dá cos(z) = 1 e sen(z) = 0,
e z = pi/2, que dá trocado), vemos que w = 1 e w = i devem ser solução
da equação, e portanto são as únicas.  Daí, z = log(1)/i = 2 k pi ou z
= log(i)/i = (2 k pi i + pi i /2)/i = 2 k pi + pi/2.

Abraços,
-- 
Bernardo

-- 
Esta mensagem foi verificada pelo sistema de antiv�rus e
 acredita-se estar livre de perigo.


=
Instru��es para entrar na lista, sair da lista e usar a lista em
http://www.mat.puc-rio.br/~obmlistas/obm-l.html
=


[obm-l] Re: [obm-l] Re: [obm-l] Função Composta

2018-05-12 Por tôpico Bernardo Freitas Paulo da Costa
Oi Ralph,

2018-05-11 20:03 GMT-03:00 Ralph Teixeira <ralp...@gmail.com>:
> (Vou supor que 0 eh natural; se nao for, apenas troque 0 por 2005 ali
> embaixo e ajeite as coisas)
>
> Primeiro: f eh injetiva. De fato, f(a)=f(b) => f(f(a))=f(f(b)) =>
> a+2005=b+2005 => a=b.
>
> Segundo: para todo n natural, f(n+2005)=f(f(f(n)))=f(n)+2005. Portanto, por
> indução, para qualquer K natural, tem-se
> f(n+K.2005)=f(n)+K.2005, ou seja, f(n+K.2005)-f(n)=K.2005.
>
> VERSÃO CURTA COM TERMINOLOGIA "MOD":
> Ou seja, mostramos que   a=b (mod 2005) => f(a)=f(b) (mod 2005).
> Agora, se f(m)=n (mod 2005), entao f(n)=f(f(m))=m+2005=m (mod 2005). Ou
> seja, f estah bem definida e eh sua propria inversa em Z_2005, o que eh
> absurdo, pois Z_2005 tem um numero impar de elementos.

Peraí, não entendi direito... se f(n) == n (mod 2005), temos uma
função que é sua própria inversa mod 2005.  Temos que excluir este
caso...

> 2018-05-11 10:42 GMT-03:00 Jeferson Almir <jefersonram...@gmail.com>:
>>
>> Como provar que nos naturais não existe a função f ( f(n) ) = n + 2005 ???
>>

Abraços,
-- 
Bernardo Freitas Paulo da Costa

-- 
Esta mensagem foi verificada pelo sistema de antiv�rus e
 acredita-se estar livre de perigo.


=
Instru��es para entrar na lista, sair da lista e usar a lista em
http://www.mat.puc-rio.br/~obmlistas/obm-l.html
=


[obm-l] Re: [obm-l] Pontos de intersecção

2018-05-02 Por tôpico Bernardo Freitas Paulo da Costa
2018-05-01 19:19 GMT-03:00 Emanuel Oliveira <emaolive...@gmail.com>:
> Um colega me pediu ajuda nesses exercicios.
>
> 1) ache os pontos de intersecção:
> a) y=arcsin(x/2) e y=2-x^2
> b)y=x*ln(x+1) e y=3x-x^2

Faça o gráfico das funções (basta um esboço, mas você pode, por
exemplo, usar o Wolfram Alpha).  Já é um bom caminho para continuar.
Aliás, "continuar" provavelmente quer dizer usar algum método
numérico, mas a própria escolha do método já deveria ser guiada pelo
gráfico, e não feita cegamente.

Abraços,
-- 
Bernardo Freitas Paulo da Costa

-- 
Esta mensagem foi verificada pelo sistema de antiv�rus e
 acredita-se estar livre de perigo.


=
Instru��es para entrar na lista, sair da lista e usar a lista em
http://www.mat.puc-rio.br/~obmlistas/obm-l.html
=


[obm-l] Re: [obm-l] Re: [obm-l] Dúvida num Enunciado

2018-04-25 Por tôpico Bernardo Freitas Paulo da Costa
2018-04-25 20:20 GMT-03:00 Pedro José <petroc...@gmail.com>:
> Boa tarde!
> Realmente o enunciado está mal feito.
>
> Se |x+3| < r, não pode ser para todo o Real. Na verdade é x pertence a |R.
>
> x^2 -10x + 9 >0  ==> x pertence a A = (-oo, 1) U (9,oo)
>
> então temos que escolher r de modo que quando resolvamos |x + 3| < r, tenha
> x num subconjunto de A
>
> x < -3 ==> x+3 < 0 ==> -x -3 < r ==> r > x+3 Se r > 4 vai ter 1=< x =<9
> atendendo |x +3| <4 + delta. Portanto x <4
> então |x+3| < 4, conferindo
> x > -3 ==> x+3 <4  ==> x<1, atende.
> se x<-3 atende por hipótese. Mas se quiser conferir. -x - 3 < 4 : -x < 7: x
>>7, mas x <-3, não tem solução.
>
> x>=- 3 ==> x+3>=0 ==> x+3 < r. Se r >=4, existirá solução em [1,9].
>
> Portanto r pertence a (0,4)

Só um detalhe: r = 4 também serve: se |x+3| < 4, temos -7 < x < 1, que
está contido em A.

A minha forma preferida de resolver este exercício é gráfica:
desenhamos o conjunto A, depois tomamos P = -3, e traçamos um
intervalo simétrico em P de maior raio possível contido em A.  Dá r <=
4 ou r < 4 (no desenho, é difícil decidir entre o estrito ou não) e
daí tem que pensar um pouco para detectar se r = 4 serve.

Abraços,
-- 
Bernardo Freitas Paulo da Costa

-- 
Esta mensagem foi verificada pelo sistema de antiv�rus e
 acredita-se estar livre de perigo.


=
Instru��es para entrar na lista, sair da lista e usar a lista em
http://www.mat.puc-rio.br/~obmlistas/obm-l.html
=


[obm-l] Re: [obm-l] Re: [obm-l] Dúvida num Enunciado

2018-04-25 Por tôpico Bernardo Freitas Paulo da Costa
2018-04-25 20:41 GMT-03:00 Claudio Buffara <claudio.buff...@gmail.com>:
> O consequente (x^2 - 10x + 9 > 0 para todo x real) é falso  (tome qualquer x
> no intervalo [1,9]).
>
> Logo, para a implicação ser verdadeira, o antecedente ( |x+3| < r ) deve ser
> falso, o que ocorre se e somente se r < 0.
>
> É mais ou menos a mesma coisa que (se 1 < 0, então 3+5 = 7), que é uma
> sentença verdadeira (Falso -> Falso é Verdadeiro).

Acho que há duas coisas.  Uma é a interpretação do enunciado.  Alguns
(eu me incluo, e o Pedro José também) vão ler como:

"Determine r > 0 tal que (para todo x real, |x+3| < r => x^2 - 10x + 9 > 0)."

A minha razão principal é porque o x aparece do lado esquerdo da
implicação, e portanto eu fico com a sensação que ele deveria também
estar quantificado.  Mas não é obrigatório.

Ao ler como você fez, a frase fica

"Determine r > 0 tal que [ |x+3| < r => (x^2 - 10x + 9 > 0 para todo x real) ]."

Nesta interpretação (que está mais próxima do texto original...), a
frase entre colchetes tem uma variável livre: o "x", que não está
quantificado.  Para enfatizar, como você mesmo separou o consequente,
ela não mudaria de valor se fosse

"Determine r > 0 tal que [ |x+3| < r => (A^2 - 10A + 9 > 0 para todo A real) ]."

Que continua com o "problema" de ter um "x" livre.  Daí, a proposição
entre colchetes tem um valor (verdadeiro/falso) que depende de x.
Assim, a frase completa "Determine r ..." também depende do valor de
x.  O problema fica bem diferente.  Primeiro, podemos simplificar o
enunciado para

"Determine r > 0 tal que [ |x+3| < r => FALSO ]."

Depois, pegando carona na sua solução, temos que ter o antecedente
falso, para que a afirmação entre colchetes seja verdadeira.  Ou seja:

"Determine r > 0 tal que [ |x+3| < r é FALSO ]."

Re-escrevendo, fica "Determine r > 0 tal que [ |x+3| >= r ].", o que
dá a solução: r <= |x+3|.  Repare que a solução está em função de x,
como esperado, já que o enunciado original também tinha um x livre.
Acho esta interpretação pouco plausível para um exercício, mas acho o
exercício de resolvê-la interessante ;-)

Abraços,
-- 
Bernardo Freitas Paulo da Costa

-- 
Esta mensagem foi verificada pelo sistema de antiv�rus e
 acredita-se estar livre de perigo.


=
Instru��es para entrar na lista, sair da lista e usar a lista em
http://www.mat.puc-rio.br/~obmlistas/obm-l.html
=


[obm-l] Re: [obm-l] Questão de derivada

2018-04-23 Por tôpico Bernardo Freitas Paulo da Costa
2018-04-22 22:36 GMT-03:00 Igor Caetano Diniz <icaetanodi...@gmail.com>:
> Boa noite,
> Gostaria de uma ajuda numa questão. Primeiro saber se pensei corretamente na
> maneira (1) e se é possível resolver como pensei também na maneira (2).
> Aí vai:
> Questão 5.3.8 do livro do Stephen Abbot, Understanding Analysis:
>
> Assuma que f é contínua em um intervalo que contém o zero e diferenciável em
> todo ponto diferente de zero. Se lim f ' (x) = L, x->0, prove que f ' (0)
> existe e é igual a L.
>
> O que pensei em fazer:
>
> Pensei em duas maneiras.
> 1)Se o limite existe em 0, então existem os limites laterais, limite a
> esquerda e limite a direita: lim x->0- f ' (x) = L e lim x->0+ f ' (x) = L.
> Lema: f ' (c) = lim f(c+h)-f(c-h)/2h = lim [ f(c+h)-f(c) +f(c) - f(c-h) ]/2h
> = 1/2 lim x->c-[f(c+h)-f(c)/h] + 1/2 lim x->c+ [f(c+h)-f(c)/h]
>
> Logo como existem esses limites laterais, existe a derivada em 0, e
> portanto, é L

Cuidado, Igor: a existência do limite [f(c+h) - f(c-h)]/2h quando h ->
0 não implica que existe a derivada.  Por exemplo, se f(x) = |x|, o
limite dá zero, mas a derivada não existe.

> 2) queria tentar fazer, usando uma sequência xn<0 com limxn = 0 e yn>0 com
> lim yn = 0 e provar que lim(f(yn)-f(xn)/(yn-xn)) = f'(0) = L. Mas sinto que
> isso é verdade e não sei provar

De novo, pelo mesmo exemplo acima, não basta provar que os limites com
um ponto de cada lado dão certo.  Acho (só acho) que se *todos* os
limites possíveis derem iguais, então a derivada existe, o que
justifica a sua abordagem, mas daí você teria que provar o enunciado
geral

"Se f é contínua, e para TODA sequência x_n < 0 < y_n, com x_n -> 0,
y_n ->0, vale que [f(y_n) - f(x_n)]/(y_n - x_n) -> L, então f é
derivável em zero, e f'(0) = L."

Abraços,
-- 
Bernardo Freitas Paulo da Costa

-- 
Esta mensagem foi verificada pelo sistema de antiv�rus e
 acredita-se estar livre de perigo.


=
Instru��es para entrar na lista, sair da lista e usar a lista em
http://www.mat.puc-rio.br/~obmlistas/obm-l.html
=


Re: [obm-l] Cantor

2018-04-18 Por tôpico Bernardo Freitas Paulo da Costa
2018-04-18 7:47 GMT-03:00 Claudio Buffara <claudio.buff...@gmail.com>:
> Agora, uma pergunta:
>
> E se fossemos fazer uma lista de todos os racionais (dízimas periódicas)
> entre 0 e 1 (por exemplo, escolhendo, quando houver ambiguidade, a versão
> que termina por ...)?
> Neste caso, o método da diagonal deveria falhar, certo, já que Q inter (0,1)
> é enumerável?
> Mas, de cara, me parece possível escolher, para todo i em N, um algarismo
> b(i) diferente de a(i,i) (= i-ésimo algarismo do número na i-ésima linha da
> lista).
> Como pode?

Adorei!  Claudio, ótimo "fora da caixa".

Abraços,
-- 
Bernardo Freitas Paulo da Costa

-- 
Esta mensagem foi verificada pelo sistema de antiv�rus e
 acredita-se estar livre de perigo.


=
Instru��es para entrar na lista, sair da lista e usar a lista em
http://www.mat.puc-rio.br/~obmlistas/obm-l.html
=


Re: [obm-l] Soma (k = 1, n) 1/P'(r_k) = 0

2018-04-16 Por tôpico Bernardo Freitas Paulo da Costa
2018-04-16 20:54 GMT-03:00 Claudio Buffara <claudio.buff...@gmail.com>:
> Resumo da ópera: ainda não temos uma demonstração elementar disso.
>
> Mas não deixa de ser interessante tentar dar uma interpretação geométrica da
> expressão para polinômios de grau baixo que tenham todas as raízes reais e
> distintas.
>
> Grau 2 é meio evidente:  as retas tangentes à parábola nas raízes têm
> inclinações de mesma magnitude e sinais opostos.

A média harmônica das inclinações é zero, o que mesmo algébrico, não
deixa de ser interessante.  E talvez "baste" achar uma visão
geométrica da média harmônica neste contexto. Outra forma de dar a
mesma equação é que "a última inclinação" é (o oposto) da média
harmônica das outras.  Como interpretar isso, tenho menos ideia
ainda...


> Grau 3 é mais interessante...
> De cara, dá margem ao problema: "Uma função polinomial de grau 3 tem raízes
> (ou será que é melhor chamar de "zeros" - funções têm zeros, quem tem raízes
> são equações...) reais e distintas: a, b e c. Se as retas tangentes ao
> gráfico da função nos pontos (a,0) e (b,0) têm inclinações m e n,
> respectivamente, determine a inclinação da reta tangente ao gráfico no ponto
> (c,0).", cuja tentativa mais óbvia de solução vai descambar num monte de
> contas (desnecessárias) envolvendo a, b e c, quando a resposta é -mn/(m+n).

Também é verdade (graças ao Douglas e o Polinômio Interpolador de
Lagrange) que a/m + b/n - c(m+n)/mn = 0, ou seja, an + bm = c(m+n), o
que dá uma relação entre m e n em função das raízes.  Observe que isso
está "certo" (de novo do ponto de vista algébrico), pois o polinômio
tem 4 coeficientes, e além das 3 raízes, precisamos de um fator
multiplicativo, que pode ser dado de várias formas: tipicamente, é o
coeficiente de mais alto grau ou o valor em um ponto particular que
não seja zero, mas agora acabamos de ver que poderia até ser a
inclinação de uma tangente a uma raiz simples! (a, b, c, m determinam
tudo!)

Aliás, isso sugere que talvez haja uma demonstração puramente
algébrica para tudo isso, contando dimensões.  O número de
coeficientes do polinômio é (n+1).  Por outro lado, dados n valores
x_i (para as raízes), e n valores m_i (para as derivadas nestes
pontos), sabemos que há n-1 equações G(X_i, M_i) = 0 para "acertar a
dimensão".  Claro que as equações devem ser simétricas nos X_i e
M_i... mas isso ainda não basta para mostrar a forma especial \sum
X_i^k / M_i = 0...  Alguém tem uma ideia?  Por exemplo, já pode ser um
bom passo mostrar que as equações são homogêneas em X e M.

Abraços,
-- 
Bernardo Freitas Paulo da Costa

-- 
Esta mensagem foi verificada pelo sistema de antiv�rus e
 acredita-se estar livre de perigo.


=
Instru��es para entrar na lista, sair da lista e usar a lista em
http://www.mat.puc-rio.br/~obmlistas/obm-l.html
=


Re: [obm-l] Soma (k = 1, n) 1/P'(r_k) = 0

2018-04-15 Por tôpico Bernardo Freitas Paulo da Costa
2018-04-15 13:09 GMT-03:00 Douglas Oliveira de Lima
<profdouglaso.del...@gmail.com>:
> Usa o polinomio de Lagrange , nao é nada obvia mesmo.

Como usa Lagrange, a fórmula segue para k = 0, 1, ... n-1
(interpolando em n pontos, vamos até grau n-1).  E é, de fato, falso
para k = n, use P(x) = (x-1)(x+1).

Além disso, mesmo para k = n-1, a demonstração por complexa não se
aplica mais (o grau dá errado...), e o mesmo polinômio serve para
mostrar que a soma não dá mais zero.

Abraços,
-- 
Bernardo Freitas Paulo da Costa

-- 
Esta mensagem foi verificada pelo sistema de antiv�rus e
 acredita-se estar livre de perigo.


=
Instru��es para entrar na lista, sair da lista e usar a lista em
http://www.mat.puc-rio.br/~obmlistas/obm-l.html
=


Re: [obm-l] Cantor

2018-04-15 Por tôpico Bernardo Freitas Paulo da Costa
2018-04-15 5:36 GMT-03:00 Luiz Antonio Rodrigues <rodrigue...@gmail.com>:
> Olá, amigos!
> Bom dia!
> Estou lendo "Matemática Discreta" da SBM e me deparei com o trecho que eu
> reproduzi abaixo.
>
>
> A principal contribuição de Cantor foi exibir casos em que não é possível
> obter uma bijeção entre dois conjuntos infinitos.
> (...)
> Seja C o conjunto de todas as sequências infinitas em que todos os termos
> são iguais a zero ou um.
> Suponhamos que fosse possível uma função f: N -> C, em que cada sequência de
> C aparecesse exatamente uma vez como imagem. Vamos construir uma sequência s
> formada por 0s e 1s (ou seja, um elemento de C) do seguinte modo: se o
> primeiro termo da sequência f(1) é zero, o primeiro termo de s é 1; senão, é
> zero. Se o segundo termo da sequência f(2) é zero, o segundo termo de s é 1;
> senão, é zero. Prosseguimos, sempre escolhendo o n-ésimo termo s(n) como
> sendo o oposto do n-ésimo termo da sequência f(n). A sequência s assim
> construída difere pelo menos na posição n de cada sequência f(n). Logo, não
> pertence à imagem de f. Mas nossa suposição era de que todos os elementos de
> C aparecessem como imagem!
> Temos, assim, uma contradição, que mostra a impossibilidade de construir uma
> bijeção de N em C.
>
> Já o reli diversas vezes. Eu "travei" na frase "A sequência s assim
> construída difere pelo menos na posição n de cada sequência f(n)."

Acho que ajuda a entender se você fizer um exemplo.  Claro que um
exemplo não prova nada, mas espero que ilumine a construção usada.

Suponha, assim, que f seja da seguinte forma:
1 -> 0100101010101
2 -> 010101010101
3 -> 11001
4 -> 
5 -> 1110111010101

Agora, vou construir a tal da sequência s, "descobrindo" o valor de
cada um dos elementos, um a um:

O primeiro elemento de s é o "oposto" do primeiro elemento de f(1).
Como o primeiro elemento de f(1) é 0, vai ser um:

s = 1

O segundo elemento de s é o oposto do segundo elemento de f(2) (que é 1):

s = 10

O terceiro elemento, oposto do terceiro de f(3), dá s = 100...
O quarto, s = 1001...
O quinto, s = 10010

Agora, repare s não pode ser f(1), nem f(2), nem f(3), nem f(4), ...
Porque o primeiro elemento de s é diferente do primeiro de f(1).  O
segundo de s, diferente do segundo de f(2). E assim por diante.
Muitas vezes, num quadro-negro, o pessoal faz a tabela que eu esbocei
acima, e envolve os elementos da "diagonal descendente", e depois cria
a sequência dos opostos.

Abraços,
-- 
Bernardo Freitas Paulo da Costa

-- 
Esta mensagem foi verificada pelo sistema de antiv�rus e
 acredita-se estar livre de perigo.


=
Instru��es para entrar na lista, sair da lista e usar a lista em
http://www.mat.puc-rio.br/~obmlistas/obm-l.html
=


[obm-l] Re: [obm-l] Re: [obm-l] Função não periódica

2018-04-14 Por tôpico Bernardo Freitas Paulo da Costa
Oi Claudio,

2018-04-14 10:54 GMT-03:00 Claudio Buffara <claudio.buff...@gmail.com>:
> f é periódica (digamos, de período T > 0).
>
> Suponhamos que g também seja periódica, digamos de período P.
>
> Para todo x, e todo k em N tal que x+kT >= 0, g(raiz(x+kT)) = f(x+kT) =
> f(x+(k+1)T) = g(raiz(x+(k+1)T)) ==>
> raiz(x+(k+1)T) - raiz(x+kT) = nP, para algum n em N.

não é verdade que, se g(x) é periódica, e g(x) = g(y), então x - y é
múltiplo do período.  Por exemplo, sin(pi/2 + a) = sin(pi/2 - a), para
todo a.

> Mas tomando k suficientemente grande, podemos fazer raiz(x+(k+1)T) -
> raiz(x+kT) tão pequeno quanto quisermos, em particular < P, o que contraria
> raiz(x+(k+1)T) - raiz(x+kT) = nP.

Intuitivamente, deve mesmo ter a ver com o que você falou sobre o
limite da diferença das raízes em PA, mas acho que é um pouco mais
complicado.  Repare que, no enunciado do Arthur, tem um "f
contínua"...

> 2018-04-12 15:55 GMT-03:00 Artur Steiner <artur.costa.stei...@gmail.com>:
>>
>> Suponhamos que f:R —> R seja contínua, periódica e não constante. Mostre
>> que g(x) = f(x^2) não é periódica.
>>
>> Artur

Abraços,
-- 
Bernardo Freitas Paulo da Costa

-- 
Esta mensagem foi verificada pelo sistema de antiv�rus e
 acredita-se estar livre de perigo.


=
Instru��es para entrar na lista, sair da lista e usar a lista em
http://www.mat.puc-rio.br/~obmlistas/obm-l.html
=


Re: [obm-l] Functional equation(ajuda)

2018-04-02 Por tôpico Bernardo Freitas Paulo da Costa
2018-04-02 8:58 GMT-03:00 Claudio Buffara <claudio.buff...@gmail.com>:
> Sem ter tido nenhuma grande ideia, eu usaria uma planilha para explorar
> ambos os problemas.
> Eu sei que, numa olimpíada de verdade, isso não seria possível. Mas isso não
> é uma olimpíada de verdade.
> E, de resto, usar planilhas pra gerar conjecturas em problemas de matemática
> pura é algo que deveria ser ensinado nas escolas desde, pelo menos, o 6o
> ano.
>
> 1) Calcular f(n) pra n pequeno (digamos, até 100 ou 200) e pra cada um dos
> 204 valores possíveis de f(1) e, com base neste "experimento" tentar gerar
> alguma conjectura.

Na verdade, o 2004 é uma boa dica que tem algo estranho.  Seja x =
f(1) (isso é de fato uma boa ideia).  Temos que f(2) = f(1+1) = x^2 +
(x^2 - 1)/2004 + 1.  (De novo, calcular para pequenos valores é uma
boa ideia em geral, não apenas para olimpíadas - e na vida real, de
fato calcular até 100 ou 200 seria um "bom mínimo".  Mas como temos a
"dica" de que o problema vem de uma olimpíada "tradicional", deve
bastar calcular muito pouco)

Ou seja, queremos encontrar x tal que x^2 - 1 = 2004k, para algum k
inteiro.  Tem a solução óbvia x = 1, que dá por recorrência f(n) = n.
Para as outras, tem que resolver a correspondente equação inteira (não
é muito difícil, basta fatorar (x+1)(x-1), tirar os fatores pares)...
Mas (com o computador) é bem fácil ver não tem solução até 204.
Aliás, eu duvido muito que isso dê certo para n = 3 depois de ter dado
certo para n = 2 ;-)  Por exemplo, tem uma solução com x = 335, que dá
f(2) = 392, mas daí f(3) deixa de ser inteiro :D)

Abraços,
-- 
Bernardo Freitas Paulo da Costa

-- 
Esta mensagem foi verificada pelo sistema de antiv�rus e
 acredita-se estar livre de perigo.


=
Instru��es para entrar na lista, sair da lista e usar a lista em
http://www.mat.puc-rio.br/~obmlistas/obm-l.html
=


[obm-l] Re: [obm-l] Re: [obm-l] Provar que uma função inteira e uniformemente contínua é um mapeamento afim

2018-03-29 Por tôpico Bernardo Freitas Paulo da Costa
2018-03-29 21:17 GMT-03:00 Claudio Buffara <claudio.buff...@gmail.com>:
> A função pode ser expressa como uma série de Taylor centrada na origem e que
> converge em todo o plano (pois é inteira, ou seja, não possui singularidades
> exceto possivelmente no infinito).
>
> Assim, f(z) = a_0 + a_1*z + a_2*z^2 + ...
>
> Mas se algum dos a_k é não-nulo para k > 1, f não poderá ser uniformemente
> contínua. Logo, f(z) = a_0 + a_1*z.

Hum, e porque, exatamente?  Acho que você gostaria de dizer que a
derivada não pode ficar limitada, e por isso f não seria uniformemente
contínua.  Mas não é imediato que "algum a_k != 0" implique que a
derivada é ilimitada em alguma sequência... afinal, os outros termos
poderiam (poderiam...) compensar, e note que você tem infinitos termos
para te ajudar a compensar...

Acho que não dá para evitar os teoremas "pesados"...

Abraços,
-- 
Bernardo Freitas Paulo da Costa

-- 
Esta mensagem foi verificada pelo sistema de antiv�rus e
 acredita-se estar livre de perigo.


=
Instru��es para entrar na lista, sair da lista e usar a lista em
http://www.mat.puc-rio.br/~obmlistas/obm-l.html
=


[obm-l] Re: [obm-l] Re: [obm-l] Problemas interessantes de análise complexa

2018-03-27 Por tôpico Bernardo Freitas Paulo da Costa
2018-03-27 13:36 GMT-03:00 Claudio Buffara <claudio.buff...@gmail.com>:
> Os problemas 1, 3 e 4 me parecem ser consequências da "rigidez" que a
> diferenciabilidade complexa impõe às funções analíticas e que, pra mim, está
> longe de ser algo intuitivo.

É, a estrutura complexa é muito impressionante.  Parte da rigidez é
puramente algébrica (como abaixo), mas existem fenômenos para os quais
eu não encontro um análogo algébrico legal (como o próximo |f| <= |g|
...)

> Por exemplo, no problema 1, se g(z) = exp(z), então a conclusão decorre do
> teorema de Liouville.
> No caso geral, temos que lidar com os zeros de g e Liouville não se aplica
> (pelo menos não diretamente).
> No entanto, se g(z) = 0, então f(z) = 0 pela desigualdade |f(z)| <= |g(z)|.
> Será que essa desigualdade garante que os zeros de g(z) são singularidades
> removíveis de f(z)/g(z)?

Sim: a forma canônica multiplicativa de f e g em torno de um zero,
mais a desigualdade, dá que a ordem de f é pelo menos igual à de g.  O
fato de haver uma ordem *inteira* de anulação é o que eu chamo de
"algebrização".

Abraços,
-- 
Bernardo Freitas Paulo da Costa

-- 
Esta mensagem foi verificada pelo sistema de antiv�rus e
 acredita-se estar livre de perigo.


=
Instru��es para entrar na lista, sair da lista e usar a lista em
http://www.mat.puc-rio.br/~obmlistas/obm-l.html
=


[obm-l] Re: [obm-l] Teorema fundamental da álgebra

2018-03-24 Por tôpico Bernardo Freitas Paulo da Costa
2018-03-24 20:13 GMT-03:00 Carlos P. <carlosp...@outlook.com.br>:
> Boa noite!

Boa noite,

> Estou estudando análise complexa e gostaria de alguns esclarecimentos sobre
> o TFA.
>
> 1) Na prova baseada no teorema de Liouville, as únicas propriedades de
> polinômios de grau >= 1 utilizadas é que são funções inteiras tais que lim z
> ---> oo p(z) = oo. Logo, o teorema aplica-se igualmente a qualquer inteira f
> tal que lim z ---> oo f(z) = oo, certo? Não está restrito a polinômios.

Exato, o teorema se aplica a qualquer função inteira tal que |f(z)| ->
oo quando |z| -> oo.  Mas olha só: esta hipótese diz que existe um R
tal que |f(z)| > 1 para |z| > R.  Daí, defina

g(w) = 1/f(1/w) no disco de raio 1/R, para w != 0.

Note que g está bem-definida, neste disco (pequeno!), e podemos
estender g(0) = 0 continuamente, logo de forma holomorfa.  A ordem do
zero de g em zero (única raiz de g no disco, por hipótese de f ser
holomorfa) é, digamos, "d".  Pelo princípio do argumento, a função g
dá "d" voltas em torno da origem dentro do disco.  Olhando agora para
f(z) no disco de raio R, ela também dá "d" voltas (na mesma direção,
porque invertemos duas vezes).  De novo pelo princípio do argumento,
isso quer dizer que f tem "d" raízes (com multiplicidade) no disco de
raio R (porque f é holomorfa, logo não tem pólos), que chamamos de
z_k.  Agora, olhando para h(z) = 1/f(z), vamos "retirar" os "d" pólos
de h(z) (= d raízes de f).  Isso dá uma função

H(z) = 1/f(z) - soma a_k/(z - z_k), onde a_k são os resíduos, que é
holomorfa (já que retiramos os pólos) no disco de raio R, mas também
fora do disco, pois os termos que adicionamos tendem a zero quando |z|
-> oo.

De novo por Liouville (agora em H(z)!), esta função é constante.
Simplificando a expressão, teremos que 1/f(z) = H + Q(z)/P(z), onde
P(z) é o produto dos fatores (z - z_k) contendo as raízes de f, e Q(z)
é o polinômio em z que aparecer.  Botando H "na fração", ficamos com
1/f(z) = QQ(z)/P(z), e assim f(z) = P(z)/QQ(z), onde QQ(z) é um
polinômio.  Como f(z) é holomorfa, QQ(z) não tem raízes, logo tem que
ser constante (pelo resultado acima!), e f é um polinômio.

Ou seja: o TFA funciona para todas as funções holomorfas em C, que
tendem a infinito no infinito.  Mas apenas os polinômios são ao mesmo
tempo holomorfos e tendem a infinito no infinito!

Um "contra-exemplo" interessante para você pensar: exp(z) é holomorfa,
mas não tem raízes.  Em que direções do plano complexo ela não tende a
infinito?

> 2) Alguém conhece uma prova do TFA que, além de mostrar a existência de
> raízes, mostre que há exatamente n raízes, contando suas ordens? Me
> informaram que há uma

Deve ser o princípio do argumento, que eu usei acima.  Ele é,
realmente, muito poderoso!
-- 
Bernardo Freitas Paulo da Costa

-- 
Esta mensagem foi verificada pelo sistema de antiv�rus e
 acredita-se estar livre de perigo.


=
Instru��es para entrar na lista, sair da lista e usar a lista em
http://www.mat.puc-rio.br/~obmlistas/obm-l.html
=


[obm-l] Re: [obm-l] Re: [obm-l] Teoria dos números

2018-03-21 Por tôpico Bernardo Freitas Paulo da Costa
2018-03-20 23:14 GMT-03:00 Anderson Torres <torres.anderson...@gmail.com>:
> Em 13 de março de 2018 20:19, Douglas Oliveira de Lima
> <profdouglaso.del...@gmail.com> escreveu:
>> Essa achei legal e estou postando.
>>
>> Resolva nos inteiros a seguinte equação:  (x + y)(y + z)(z + x)/2 + (x + y +
>> z)3 = 1 – xyz .
>>
>
> Substituição mágica: x=-a+b+c, y=a-b+c, z=a+b-c. Com isso, x+y=2c, 
> x+y+z=a+b+c e
>
> 4abc + (a+b+c)^3 + (-a+b+c)(a-b+c)(a+b-c) = 1
>
> Usando polinômios simétricos,
>
> 4(a+b+c)(ab+ac+bc) - 4abc = 1
>
> Agora estou confuso...

Note que a,b,c não precisam mais ser inteiros, podem ser inteiros
divididos por 2 (se não me engano)

Abraços,
-- 
Bernardo Freitas Paulo da Costa

-- 
Esta mensagem foi verificada pelo sistema de antiv�rus e
 acredita-se estar livre de perigo.


=
Instru��es para entrar na lista, sair da lista e usar a lista em
http://www.mat.puc-rio.br/~obmlistas/obm-l.html
=


Re: [obm-l] Limite

2018-03-19 Por tôpico Bernardo Freitas Paulo da Costa
2018-03-19 12:27 GMT-03:00 Vanderlei Nemitz <vanderma...@gmail.com>:
> Bom dia!
> Eu resolvi o limite a seguir de um modo muito complicado e encontrei 1/e.
>
> Alguém conhece alguma solução?
>
> lim [n!/n^n]^(1/n), quando n tende ao infinito.

Eu imagino que seja para usar a equivalência entre o teste da raiz e o
teste da razão.

Abraços,
-- 
Bernardo Freitas Paulo da Costa

-- 
Esta mensagem foi verificada pelo sistema de antiv�rus e
 acredita-se estar livre de perigo.


=
Instru��es para entrar na lista, sair da lista e usar a lista em
http://www.mat.puc-rio.br/~obmlistas/obm-l.html
=


Re: [obm-l] Como calcular?

2018-03-01 Por tôpico Bernardo Freitas Paulo da Costa
2018-03-01 0:56 GMT-03:00 Gabriel Tostes <gtos...@icloud.com>:
> Define a sequencia A_(n+1)= [ (A_n)^2 - 1 ] / n (1)
> Então A_2= sqrt(1+2A3)=sqrt(1+2(sqrt(1+3A4))... Realimentando sempre 
> (substituindo A_n=sqrt(1+ nA_n+1)
> vemos que A2 se iguala a x se lim n->oo da raiz 2^(n-2) de An é 0.

Eu não entendi esta afirmação "A2 se iguala a x se lim ... = 0".  Como
você mostra isso?  Além do mais, a sequência de raizes podia
(podia...) tender a infinito.  Acho que também tem que mostrar que não
é o caso.  Enfim, o que você escreveu (pode ser que você queira dizer
outra coisa) é que "se o limite é zero, então A2 é finito", mas o que
você precisa (para o argumento abaixo) é "se A2 é finito, então o
limite é zero".

> Seja An=n+1 + Bn. Bn outra sequencia.

Realmente, essa transformação é mágica.  Eu chutei o limite (usando um
computador) e daí calculei os outros termos, vi An = n+1, e fui provar
que dava.  O que eu usei foi uma sequência dupla, T(n,m) = raiz(1 +
n*raiz(1 + (n+1)*raiz(1 +  raiz(1 + m) ... ))).  Claro que
T(n,m+1) > T(n,m), portanto lim T(n,m) existe (ou é infinito).  E para
provar que não é infinito eu usei que o limite deveria dar T(n,inf) =
n+1, e provei que T(n,m) < n+1 para todo m...

> Então, de 1:
> n+2+B_(n+1)=n+2 + 2Bn + ( Bn^2 + 2Bn)/n -> B_(n+1) >= 2Bn, uma inducao 
> simples traz que:
> Bn>=2^(n-2).B2
> Entao o limite quando n vai para o infinito da raiz 2^(n-2) de An eh igual a 
> B2, ou seja, B2=0 e
> X= A2=3

-- 
Bernardo Freitas Paulo da Costa

-- 
Esta mensagem foi verificada pelo sistema de antiv�rus e
 acredita-se estar livre de perigo.


=
Instru��es para entrar na lista, sair da lista e usar a lista em
http://www.mat.puc-rio.br/~obmlistas/obm-l.html
=


Re: [obm-l] soma de quadrados

2018-02-28 Por tôpico Bernardo Freitas Paulo da Costa
2018-02-28 22:01 GMT-03:00 marcone augusto araújo borges
:
> Seja a sequência
>
> 3^2 + 4^2 = 5^2
> 3^2 + 4^2 + 12^2 = 13^2
> 3^2 + 4^2 + 12^2 + 84^2 = 85^2
>.
>.
>.
> A soma de n quadrados é um quadrado
> Existe uma ´´lei de formação´´ ou uma recorrência para determinar
> uma soma dessas para, digamos, n = 10 ou n = 30 ou n = 100, ...

Vou dar (um) próximo termo.  Não é, necessariamente, o menor, nem o
melhor, mas ele tem uma "lei de formação" fácil.

3^2 + 4^2 + 12^2 + 84^2 + 204^2 = 221^2


A sequência que eu obtive tem crescimento "exponencial", ou seja, o
n-ésimo termo é maior do que 2^n.  Seria interessante saber se existe
uma sequência de crescimento polinomial...

Abraços,
-- 
Bernardo

-- 
Esta mensagem foi verificada pelo sistema de antiv�rus e
 acredita-se estar livre de perigo.


=
Instru��es para entrar na lista, sair da lista e usar a lista em
http://www.mat.puc-rio.br/~obmlistas/obm-l.html
=


Re: [obm-l] determine all pair of integers (x,y) such that

2018-02-24 Por tôpico Bernardo Freitas Paulo da Costa
2018-02-24 9:47 GMT-03:00 Luís Lopes <qed_te...@hotmail.com>:
> 1 + 2^x + 2^(2x+1) = y^2
>
> Recebi o problema acima de um outro grupo.
>
> Como resolver ?

Oi Luis,

Eu começaria retirando as coisas "óbvias": mostre que tem uma solução
com x = 0, que x = -1 não dá, e que para x < -1 também não dá porque o
lado esquerdo fica entre 1 e 2 estritamente.  Também testando, x = 1,
2, 3 dão 11, 37, 137, que não são quadrados.

Agora vamos fatorar (o que for possível) e depois separar em casos.  É
uma solução meio força-bruta.





Deixo um espaço aqui para quem quiser continuar pensando (e achar algo
menos apelativo).





- Ponha z = 2^x.  Temos z(1 + 2z) = y^2 - 1 = (y-1)(y+1), e z é uma
potência "grande" de 2, e (1 + 2z) é ímpar.
- Logo, o produto (y-1)(y+1) é par, pelo menos um dos fatores é par,
logo ambos os fatores são pares.
- Nesta fatoração, exatamente um dos termos (y-1) ou (y+1) contém uma
potência grande de 2 (grande quer dizer "maior do que 2^2"), o outro
será 2*ímpar.
- Logo, y + d = 2^m * k, com k ímpar, m >= 2 (potência grande), e d =
+1 ou -1 (para escolher qual dos fatores vai ser).  Daí deduzimos que
o outro fator, y - d, é igual a 2^m * k - 2*d = 2(2^(m-1)*k - d), que
é 2*ímpar porque m >= 2.

Juntando: 2^x * (1 + 2*2^x) = 2^m * k * 2 * (2^(m-1) * k - d) =
2^(m+1) * k * (2^(m-1) * k - d)
- Olhando para as potências de 2, temos x = m+1 >= 3.

Vamos olhar para a parte ímpar.  Para simplificar a notação, vou usar
n = m-1 >= 1
Do lado esquerdo, temos 1 + 2*2^x = 1 + 4*2^m = 1 + 8*2^n
Do lado direito, temos k * (2^n * k - d)
Dividindo por 2^n em ambos os lados, ficamos com 8 + 1/2^n = k^2 - kd/2^n

Se d = +1, esta equação dá 8 = k^2 - (k+1)/2^n, o que mostra que k >= 3.
Para k = 3, temos 8 = 9 - 1, logo (k+1)/2^n = 1, o que dá 2^n = 4, n =
2.  Isso corresponde à solução x = 4, y = 23.  (quase não acreditei,
mas é verdade!  Isso dá um pouco de confiança que eu não errei muitas
contas aí pra trás!)
Como k^2 - (k+1)/2^n é uma função crescente de k para k > 3 (lembre
que n >= 1), o lado direito vai ser sempre maior do que 5^2 - 6/2 =
18.

Se d = -1, esta equação é 8 = k^2 + (k+1)/2^n, o que dá k < 3.  Só
poderia ser k = 1 (pois é ímpar), e o maior valor possível é 1 +
(1+1)/2 = 2, muito longe.
Então não há outra solução.

Ufa!!

Abraços,
-- 
Bernardo Freitas Paulo da Costa

-- 
Esta mensagem foi verificada pelo sistema de antiv�rus e
 acredita-se estar livre de perigo.


=
Instru��es para entrar na lista, sair da lista e usar a lista em
http://www.mat.puc-rio.br/~obmlistas/obm-l.html
=


Re: [obm-l] Cotangente irracional

2018-02-06 Por tôpico Bernardo Freitas Paulo da Costa
2018-02-06 20:46 GMT-02:00 Israel Meireles Chrisostomo
<israelmchrisost...@gmail.com>:
> Se cot(x) é irracional e y é um dado real, então ou cot((x+y)/2) ou
> cot((x-y)/2) é irracional. Alguém poderia me dar uma solução mais simples do
> que eu apresentei para esse problema?
> Eis o link para a solução(último problema, página 256 ):
> https://www.sharelatex.com/read/mptcvdxqqfgq

Cara, é MUITO mais simples...  Vou dar um espaço aqui se mais alguém
da lista quiser pensar numa solução.




[Dica: pense numa contrapositiva]



Sejam a,b dois ângulos. tan(a+b) = [ tan(a) + tan(b) ] / [1 - tan a
tan b].  Note que cot() é racional <=> tan() é rScional.  Suponha por
absurdo que ambos u = tan((x+y)/2) e v = tan((x-y)/2) sejam racionais.
Daí, tan(x) = tan( (x+y)/2 + (x-y)/2 ) = [u + v]/[1 - uv] também é
racional.

Abraços,
-- 
Bernardo Freitas Paulo da Costa

-- 
Esta mensagem foi verificada pelo sistema de antiv�rus e
 acredita-se estar livre de perigo.


=
Instru��es para entrar na lista, sair da lista e usar a lista em
http://www.mat.puc-rio.br/~obmlistas/obm-l.html
=


[obm-l] Re: [obm-l] Re: [obm-l] Re: [obm-l] Re: [obm-l] Re: [obm-l] Re: [obm-l] Questão de Cardinalidade

2018-01-16 Por tôpico Bernardo Freitas Paulo da Costa
2018-01-16 14:11 GMT-02:00 Igor Caetano Diniz <icaetanodi...@gmail.com>:
> Fala Bernardo, tudo certo?
> Mas sera que eu conseguiria provar que esses números não seriam uma
> quantidade enumeravel de pontos entre 0 e 1 e, então, como é enumeravel, eu
> consigo pegar uma quantidade enumeravel em P(N) para esses pontos.

Sim, de fato são enumeráveis (é um exercício legal provar isto).  Dá
um pouco mais de trabalho "modificar" as bijeções para corrigir o que
está acontecendo nestes pontos

> Acha que
> seria ruim?

Não digo que seja ruim, só acho que é "trabalho demais" quando você
poderia ir por um caminho mais simples ;-)

Abraços,
-- 
Bernardo Freitas Paulo da Costa

-- 
Esta mensagem foi verificada pelo sistema de antiv�rus e
 acredita-se estar livre de perigo.


=
Instru��es para entrar na lista, sair da lista e usar a lista em
http://www.mat.puc-rio.br/~obmlistas/obm-l.html
=


[obm-l] Re: [obm-l] Re: [obm-l] Re: [obm-l] Re: [obm-l] Questão de Cardinalidade

2018-01-16 Por tôpico Bernardo Freitas Paulo da Costa
2018-01-16 1:10 GMT-02:00 Anderson Torres <torres.anderson...@gmail.com>:
> Eu na verdade pensei ao contrário:
>
> Começamos com o conjunto de todos os subconjuntos de N. Cada conjunto
> será representado por uma string infinita de zeros e unzes, da
> seguinte forma: Se o conjunto contiver o natural x, o x-ésimo
> caractere desta string será 1; caso contrário, será 0.
>
> Botando zero-vírgula na frente, obtemos um número real escrito em base
> 2, contido no intervalo [0,1] (para efeito de completude do argumento,
> admitiremos strings infinitas de 1zes).
>
> Para cada real em [0,1], bastaria escrever na base 2 e criar um
> conjunto a partir daí, seguindo os passos acima (se o X-esimo dígito é
> 1, escolhe X, caso contrário, despreza X).
>
> Isso prova que existe uma bijeção entre o conjunto das partes de N e o
> intervalo [0,1].

Acho que tanto a sua demonstração como a do Sávio têm um problema:

0,0111... = 0.1...

Isso quer dizer que o conjunto {0} e o conjunto {1,2,3,...} são
enviados no mesmo número real (conhecido como 1/2, ou 0.5 em decimal).

Eu sempre acho muita "forçação de barra" tentar exibir uma bijeção.
99% das vezes, é mais esforço do que precisa, sem ganhar muito
entendimento.  Ou, como neste caso, papa-se uma mosca...  Minha
sugestão é exibir uma sobrejeção de P(IN) em IR, e depois uma
sobrejeção de IR em P(IN).  A primeira está garantida, pois basta
compor a construção do número binário em [0,1] com qualquer sobrejeção
deste conjunto em R.  Uma sobrejeção simples é mandar 0 e 1 "pra
qualquer lugar", e depois usar uma bijeção de (0,1) em IR.

Deixo para vocês pensarem como fazer para exibir uma sobrejeção de IR
nas partes de IN.  Dica: IR contém [0,1) e [1,2).

Abraços,
-- 
Bernardo Freitas Paulo da Costa

-- 
Esta mensagem foi verificada pelo sistema de antiv�rus e
 acredita-se estar livre de perigo.


=
Instru��es para entrar na lista, sair da lista e usar a lista em
http://www.mat.puc-rio.br/~obmlistas/obm-l.html
=


[obm-l] Re: [obm-l] Re: [obm-l] Teoria dos números

2017-11-21 Por tôpico Bernardo Freitas Paulo da Costa
2017-11-21 22:41 GMT-02:00 Anderson Torres <torres.anderson...@gmail.com>:
> Que treta... Bem, a ideia seria descobrir a potência de dez que deixa
> resto um módulo 3^2002, e daí realizar a divisão longa
> ((10^k-1)/2002)...
>
> Em 21 de novembro de 2017 17:13, Vinícius Raimundo
> <vini.raimu...@gmail.com> escreveu:
>> Encontre o período na representação decimal de 1/3^2002

Eu acho que por "o período" o enunciado quer dizer o número de dígitos
no período, (assim 1/3 = 0.3... tem período 1, 1/7 =
0.142857142... tem período 6, etc).

E neste caso uma solução por recorrência mostra que o período é
3^{n-2} para n >= 2 (se não me falha a memória).  Se for isso, basta
mostrar que 10^{3^n} - 1 é divisível por 3^{n+2}, mas não por 3^{n+3}.
Com n=0, isso é 10^1 - 1 é divisível por 9 = 3^2, o passo de indução
você usa (a-1)^ = a^3 - 3a^2 + 3a - 1.  Agora, calcular a parte
periódica, vai demorar... Se eu acertei as contas, tem "só" 3^2000
dígitos, que é muito mais do que o número de átomos no universo (por
volta de 10^{80}, de novo de memória) :)

Abraços,
-- 
Bernardo Freitas Paulo da Costa

-- 
Esta mensagem foi verificada pelo sistema de antiv�rus e
 acredita-se estar livre de perigo.


=
Instru��es para entrar na lista, sair da lista e usar a lista em
http://www.mat.puc-rio.br/~obmlistas/obm-l.html
=


Re: [obm-l] cadeia de logaritmos

2017-11-20 Por tôpico Bernardo Freitas Paulo da Costa
Oi Luis,

como você mandou no outro email, mas o enunciado e a minha sugestão
estão aqui, vou continuar aqui.  Espero que você não tenha problemas
em responder aqui...

2017-11-07 14:05 GMT-02:00 Bernardo Freitas Paulo da Costa
<bernardo...@gmail.com>:
> 2017-11-04 10:00 GMT-02:00 Luís Lopes <qed_te...@hotmail.com>:
>> Sauda,c~oes,
>>
>>
>> Considere o número N = ((100¹°°)¹°°)¨¨¹°° (ou seja, 100 elevado a 100,
>> elevado a 100, ...), onde o número 100 aparece 100 vezes (incluindo a base).
>> Seja a sequência definida como:
>>
>> a1 = log N
>> a2 = log (a1)
>> a3 = log (a2)
>> ...
>> a99 = log (a98)
>> a100 = log (a99)
>>
>> Pode-se afirmar que a99 + a100 é igual a:

Das suas contas na outra mensagem, depreendo, também, que log deve ser
entendido em base 10.  Isso é um pouco mais chato do que base e (para
fazer análise, que é a minha solução) mas acho que não vai atrapalhar.

> Vou dar uma sugestão: prove que
>
> C + eps < a_i < C + 2eps
>
> implica que
>
> log(C) + eps < a_{i+1} < log(C) + 2eps
>
> se C for "grande" e eps for "pequeno".

Ops, me enganei na formulação exata.  Vou corrigir, mas antes disso
deixa eu introduzir uma notação:

b0 = 1
b1 = 100
b2 = 100^{100}
...
b100 = 100^{b99} = N

Agora, faça algumas iterações, para perceber o que está acontecendo:

a1 = log(N) = b99 * log(100) = 2 b99
a2 = log(a1) = log(2) + 2 b98 = 2 b98 (1 + eps98)
a3 = log(a2) = log(2) + 2 b97 + log(1 + eps98) = 2 b97(1 + eps97)

O problema que temos aqui é que eps97 é *bem feio*.  E só vai piorar
conforme iteramos.  Portanto, precisamos de um resultado parecido como
o que eu dei acima (só que certo, o que está acima é falso!)

Um dedinho de análise: ln(1+x) < x, para qualquer x.  Vamos usar
sempre para x positivo, mas vale sempre, por concavidade do ln.  Daí,
log(1+x) = ln(1+x)/ln(10) < x/ln(10)

Já percebemos que a cada log() que usarmos, vai "sair" um log(2),
então vou usar eps = log(2).  Quero então mostrar, por indução, que

2 b_{100-k} + log(2) <= a_k <= 2 b_{100-k} + 2 log(2)

O lado esquerdo é fácil: a_{k+1} = log(a_k) >= log(2 b_{100-k}) =
log(2) + b_{100-k-1}

Vejamos, agora, o lado direito.  Fatorando como antes, a_k <= 2
b_{100-k} (1 + log(2)/b_{100-k}). Logo

a_{k+1} <= log(2) + b_{100 - k - 1} + log(1 + log(2)/b_{100-k})

Como log(1+x) < x/ln(10), basta mostrar que log(2)/b_{100-k} < log(2)
* ln(10), ou seja, b_{100-k} ln(10) > 1, o que é verdade para todo k
<= 99, já que b1 = 100 > 1, e os b_k são cada vez maiores.

Isso dá 2*1 + log(2) <= a100 <= 2*1 + 2 log(2) e 2*100 + log(2) <= a99
<= 2*100 + 2 log(2), logo a soma está entre 202 + 2log(2) e 202 +
4log(2), ou seja, entre 202.6 e 203.2.  Como eu aposto que a
estimativa de 2log(2) é MUITO ruim, e que o valor deve estar mais
próximo da parte inferior, eu acho que a resposta certa será 202.6,
que não está nas alternativas.  Mas por outro lado para ser 202.3,
seria necessário não haver nenhum dos log(2), e estes TÊM que estar,
por conta do 2* que aparece com log(100) = 2.

Espero que ajude...

Abraços,
-- 
Bernardo Freitas Paulo da Costa

-- 
Esta mensagem foi verificada pelo sistema de antiv�rus e
 acredita-se estar livre de perigo.


=
Instru��es para entrar na lista, sair da lista e usar a lista em
http://www.mat.puc-rio.br/~obmlistas/obm-l.html
=


Re: [obm-l] cadeia de logaritmos

2017-11-07 Por tôpico Bernardo Freitas Paulo da Costa
2017-11-04 10:00 GMT-02:00 Luís Lopes <qed_te...@hotmail.com>:
> Sauda,c~oes,
>
>
> Bom dia.
>
>
> Não consegui resolver a questão abaixo. Como fazer ?
>
>
> Abraços,
>
> Luís
>
>
>
> Considere o número N = ((100¹°°)¹°°)¨¨¹°° (ou seja, 100 elevado a 100,
> elevado a 100, ...), onde o número 100 aparece 100 vezes (incluindo a base).
> Seja a sequência definida como:
>
> a1 = log N
> a2 = log (a1)
> a3 = log (a2)
> ...
> a99 = log (a98)
> a100 = log (a99)
>
> Pode-se afirmar que a99 + a100 é igual a:

Vou dar uma sugestão: prove que

C + eps < a_i < C + 2eps

implica que

log(C) + eps < a_{i+1} < log(C) + 2eps

se C for "grande" e eps for "pequeno".  Daí, mostre que ser
(relativamente) "pequeno" e "grande" é verdade por indução ;-)

Abraços,
-- 
Bernardo Freitas Paulo da Costa

-- 
Esta mensagem foi verificada pelo sistema de antiv�rus e
 acredita-se estar livre de perigo.


=
Instru��es para entrar na lista, sair da lista e usar a lista em
http://www.mat.puc-rio.br/~obmlistas/obm-l.html
=


[obm-l] Re: [obm-l] Problema difícil.

2017-09-12 Por tôpico Bernardo Freitas Paulo da Costa
2017-09-12 17:51 GMT-03:00 Douglas Oliveira de Lima
<profdouglaso.del...@gmail.com>:
> Considere a sequência de números 1,2,3,4,5,...,2017.
> E uma certa ordenação deles a1, a2, a3, ..., a2017.
> Agora multiplique respectivamente os números das duas sequencias
> determinando assim uma nova sequência 1.a1, 2.a2, 3.a3, ..., 2017.a2017.
>
> Qual o menor valor que o maior produto da última sequência pode assumir?

Esse problema não é tão difícil quanto parece.  O que você tentou fazer?

Abraços,
-- 
Bernardo Freitas Paulo da Costa

-- 
Esta mensagem foi verificada pelo sistema de antiv�rus e
 acredita-se estar livre de perigo.


=
Instru��es para entrar na lista, sair da lista e usar a lista em
http://www.mat.puc-rio.br/~obmlistas/obm-l.html
=


[obm-l] Re: [obm-l] Dúvida em como resolver um sistema de eq lineares em N

2017-09-07 Por tôpico Bernardo Freitas Paulo da Costa
2017-09-07 3:33 GMT-03:00 Michel Torres <mevtor...@gmail.com>:
> Olá Pessoal
>
> Como ataco esse problema?
>
> possuo duas equções lineares
>
> eq1 = aX+bY+cz+dW
>
> eq2 = eX+fY+gZ+hW
>
> a,b,c,d,e,f,g,h valores conhecidos
>
> Restrição X+Y+Z+W = 6 e X,Y,Z,W assumem somente valores inteiros
> {0,1,2,3,4,5,6}
>
> Quero encontrar para quais valores de X,Y,Z,W  obtenho um valor máximo para
> eq1 e ao mesmo tempo um valor mínimo para eq2

Bom, se você nunca estudou programação linear, esta seria a primeira
parte.  O seu problema parece um PL, com variáveis inteiras.  Eu digo
"parece" porque você deseja ao mesmo tempo maximizar eq1 e minimizar
eq2, como você disse.  E isso, até onde eu entendo, é um problema mal
posto.  Vou tentar dar um exemplo completo:

Suponha que e>f>g>h>0; nestas condições, (0,0,0,6) minimiza eq2, e é o
único que realiza o mínimo
Suponha que (a,b,c,d) = (2,2,1,1); de novo, observe que (X,Y,Z,W) =
(2,2,1,1) maximiza a eq1 e também é o único.

Como fazer para escolher entre estes valores?  Percebe que parece que
"falta um critério de desempate"?  Talvez você queira um valor que
maximiza eq1, e dentre todos os possíveis, um que minimize eq2.  (Isso
daria a solução (2,2,1,1)).  Talvez seja o contrário: é mais
importante minimizar eq2, e você dentre todos que dão o mesmo mínimo,
você escolhe o que maximiza eq1.  E poderia ser outra coisa também...
(só para te dar mais coisas para pesquisar, esta abordagem é conhecida
como "otimização multi-objetivo", e um dos primeiros tópicos é sobre
como transformar isso num problema bem-posto, com solução clara, etc)

Abraços,
-- 
Bernardo Freitas Paulo da Costa

-- 
Esta mensagem foi verificada pelo sistema de antiv�rus e
 acredita-se estar livre de perigo.


=
Instru��es para entrar na lista, sair da lista e usar a lista em
http://www.mat.puc-rio.br/~obmlistas/obm-l.html
=


Re: [obm-l] Passageiros em fila (probabilidade)

2017-08-31 Por tôpico Bernardo Freitas Paulo da Costa
On Wed, Aug 30, 2017 at 2:30 PM, Gabriel Tostes <gtos...@icloud.com> wrote:
> Me mandaram esse problema. Primeiro eu fiz tbm com induçao e etc. Mas como o
> resultado era mto bonito fui pensar de outra maneira, mais rapida. Vamos la:
>
> No decorrer das pessoas sentando, a ultima nao sentará na cadeira dela
> somente se uma pessoa ja a tenha ocupado. Porem, para a pessoa que for
> ocupar a cadeira do ultimo passageiro, damos uma opcao a ela: ou sente na
> primeira cadeira ou na ultima. No primeiro caso a ultima pessoa sentará na
> cadeira dela, no segundo nao. Em ambos os casos as cadeiras de todas outras
> pessoas vao estar definidas e, logo, tem uma bijeicao entre os arranjos em
> que a ultima pessoa senta na cadeira dela ou nao.  A probabilidade eh,
> entao, 1/2.

Ter uma prova bijetiva seria legal.  Mas eu confesso que não entendi o
seu argumento.  Eu acho que você quis dizer o seguinte:
- Considere todas as possíveis ocorrências do processo das pessoas
sentando, chame este conjunto de X.
- Um elemento x de X é uma correspondência das pessoas com os assentos
em que elas de fato ficaram.
- O conjunto X não é equiprovável, pois se por exemplo o "louco" se
sentar na sua cadeira certa (com probabilidade 1/k, onde k é o número
de pessoas depois dele na fila), só há UM evento em X que corresponde
a isso.  Por outro lado, se ele se sentar na cadeira de outra pessoa
(que não seja a última), a probabilidade dos eventos nestas
circunstâncias é menor, pois tem que usar um "princípio
multiplicativo" para cada vez que uma pessoa tiver que escolher uma
cadeira, se a sua estiver ocupada.
- Suponha que a última pessoa não se sentou no seu assento marcado.
Isto define um subconjunto Y de X.  Para cada evento y de Y, existe
uma pessoa p que se sentou na cadeira do último.  Esta pessoa tem j
pessoas depois dela.
- Aqui eu começo a não entender... o que quer dizer a sua "opção" de
"sentar na primeira cadeira"??? Esta tal primeira cadeira já não está
ocupada?  Ou você quis dizer "o cara, em vez disso, vai se sentar na
cadeira que o último se sentou"?
- De qualquer forma, ao alterar a decisão da pessoa p, vai ocorrer
também uma mudança das probabilidades dos eventos (pois a "nova"
cadeira que ele escolheu de fato pertencia a outra pessoa, e esta
outra pessoa agora vai ter que escolher uma outra cadeira para sentar,
...)

Assim, eu não entendi direito como você constrói a bijeção, e mesmo
que houvesse uma bijeção, você teria que provar que os eventos postos
em bijeção tem a mesma probabilidade, o que não é imediato, já que os
eventos em X não são equiprováveis.

Abraços,
-- 
Bernardo Freitas Paulo da Costa

-- 
Esta mensagem foi verificada pelo sistema de antiv�rus e
 acredita-se estar livre de perigo.


=
Instru��es para entrar na lista, sair da lista e usar a lista em
http://www.mat.puc-rio.br/~obmlistas/obm-l.html
=


Re: [obm-l] Garrafa de Klein

2017-08-14 Por tôpico Bernardo Freitas Paulo da Costa
2017-08-14 12:13 GMT-03:00 Carlos Nehab <carlos.ne...@gmail.com>:
> Facilmente.

Eu digo que é impossível, mas depende do que você chama "construir".
A garrafa de Klein é uma superfície não-orientável.  O que se vende
por aí (ou que até dá pra fazer em casa) é uma representação "quase
fiel" da garrafa de Klein.  A verdadeira garrafa não possui
auto-interseção, ao contrário dos modelos que podem ser realizados em
R^3.

Só para (tentar) dar uma analogia: imagine que você tem uma corda e
faz um nó nela.  Você só consegue fazer o nó porque você pegou este
objeto 1-dimensional (a corda) e usou todo o espaço de R^3 para dar o
nó.  Você nem consegue fazer o nó "ficar" no plano, porque uma parte
vai passar por cima e outra por baixo, AINDA usando a "folga" de R^3.
Para "caber" no plano, só se você "soldar" a parte que está passando
por cima com a que passa por baixo, mas daí você mudou a corda.  O
mesmo problema acontece com a garrafa de Klein: a versão que a gente
consegue fazer em R^3 tem alguns pontos onde ela foi "soldada nela
mesma", o que não é o caso para a verdadeira garrafa de Klein.

Para os amadores de termos técnicos: a garrafa de Klein pode ser
imersa em R^3 (o que permite auto-interseções, como o que acontece com
o nó em R^2), e mergulhada em R^4 (o que preserva toda a garrafa, em
particular, não gera auto-interseções).  (para a definição dos termos,
veja https://en.wikipedia.org/wiki/Submanifold, e para as afirmações
particulares sobre a garrafa o primeiro parágrafo de
https://en.wikipedia.org/wiki/Klein_bottle#Properties - não sei se o
resto desta seção é realmente uma boa idéia ler se você não faz
geometria diferencial pesada :D)

> Em 13/08/2017 20:17, "Luiz Antonio Rodrigues" <rodrigue...@gmail.com>
> escreveu:
>>
>> Olá, pessoal!
>> Boa noite!
>> Eu tenho uma dúvida desde os tempos da faculdade... Uma garrafa de Klein
>> pode ser construída? Eu consultei a Internet e me confundi ainda mais...
>> Um abraço para todos!
>> Luiz


Abraços,
-- 
Bernardo Freitas Paulo da Costa

-- 
Esta mensagem foi verificada pelo sistema de antiv�rus e
 acredita-se estar livre de perigo.


=
Instru��es para entrar na lista, sair da lista e usar a lista em
http://www.mat.puc-rio.br/~obmlistas/obm-l.html
=


[obm-l] Re: [obm-l] Re: [obm-l] Re: [obm-l] Questão de teoria numérica

2017-08-01 Por tôpico Bernardo Freitas Paulo da Costa
Em 01/08/2017 08:14, "Pedro Cardoso" <mr.pedrocard...@gmail.com> escreveu:
>
> Obrigado! Era exatamente isso que a questão anterior sugeria, usar o 
> princípio da casa dos pombos.
> Uma coisa que percebi na sua dsmonstração é que o número encontrado 
> terminaria em 0s, mas como nenhum multiplo de 2017 também é multiplo de 10 
> (2017 é primo) então também existe um multiplo de 2017 com apenas 1s!

A conclusão está certa (existe um múltiplo de 2017 só com 1s), mas a
justificativa está errada: 20170 é múltiplo de 2017 e de 10.  Tem a
ver com primalidade, mas não é bem o que você escreveu.

Abraços,
-- 
Bernardo Freitas Paulo da Costa

-- 
Esta mensagem foi verificada pelo sistema de antiv�rus e
 acredita-se estar livre de perigo.


=
Instru��es para entrar na lista, sair da lista e usar a lista em
http://www.mat.puc-rio.br/~obmlistas/obm-l.html
=


[obm-l] Re: [obm-l] Re: [obm-l] Re: [obm-l] Dúvida em uma solução (conjunto denso)

2017-07-10 Por tôpico Bernardo Freitas Paulo da Costa
2017-07-10 18:56 GMT+03:00 Antonio Carlos <ac6945...@gmail.com>:
> Entendi. Muito obrigado, Pedro!

Tem um problema muito sério, que os logs são diferentes...

log_2 3 = log(3)/log(2) = 1.5849625007211563
log_3 6 = log(6)/log(3) = 1.6309297535714573

Mas o problema está, provavelmente, na primeira hipótese (que ela
também é falsa).  A demonstração por densidade está certa, e talvez
seja no meio de um raciocínio por absurdo, mas sei lá...
-- 
Bernardo Freitas Paulo da Costa

-- 
Esta mensagem foi verificada pelo sistema de antiv�rus e
 acredita-se estar livre de perigo.


=
Instru��es para entrar na lista, sair da lista e usar a lista em
http://www.mat.puc-rio.br/~obmlistas/obm-l.html
=


Re: [obm-l] Problema estranho

2017-07-09 Por tôpico Bernardo Freitas Paulo da Costa
Não pensei muito, mas acho que você deveria tentar provar os casos n=1
e n=2 "no braço" para ter a intuição.  E, na verdade, o enunciado
deveria ser: dados a_1, a_2, ... a_{2n+1} números reais, não
necessariamente distintos, tais que, para cada escolha de 2n dentre
eles, é possível separar em dois grupos de n cada, com a mesma soma.
(evitando falar de conjuntos, você pode ter à vontade os elementos
repetidos).

Assim, o caso n=1 fica: temos a_1, a_2, a_3.  Tomando os elementos
a_1, a_2, é possĩvel separar em dois grupos de um elemento, com a soma
igual.  Logo a_1 = a_2.  Por simetria, a_1 = a_3, e acabou.  Para n=2,
dá mais trabalho.

2017-07-08 23:20 GMT+03:00 Otávio Araújo <otavio17.ara...@gmail.com>:
> Galera, queria que alguém pudesse resolver essa questão pra mim ( passei 
> muito tempo nela já kkk):
> " Seja n um natural positivo e A um conjunto de 2n+1 números reais, não 
> necessariamente distintos, com a seguinte propriedade:
> - Todo subconjunto de A com 2n elementos pode ser particionado em dois 
> conjuntos de n elementos tais que a soma dos elementos de cada um desses dois 
> conjuntos de n elementos são iguais.
>Prove que todos os elementos de A são iguais."
>
>
>
>
>
>
>
> --
> Esta mensagem foi verificada pelo sistema de antivírus e
>  acredita-se estar livre de perigo.
>
>
> =
> Instruções para entrar na lista, sair da lista e usar a lista em
> http://www.mat.puc-rio.br/~obmlistas/obm-l.html
> =========



-- 
Bernardo Freitas Paulo da Costa

-- 
Esta mensagem foi verificada pelo sistema de antiv�rus e
 acredita-se estar livre de perigo.


=
Instru��es para entrar na lista, sair da lista e usar a lista em
http://www.mat.puc-rio.br/~obmlistas/obm-l.html
=


Re: [obm-l] Integral complexa

2017-07-04 Por tôpico Bernardo Freitas Paulo da Costa
2017-06-26 3:06 GMT+02:00 Artur Costa Steiner <steinerar...@gmail.com>:
> Esse me parece interessante

+1 ;-)

Dica: estude a função z^n(z - 2).

> Sejam P_n o polinômio definido nos complexos por P_n(z) = (z^n) (z - 2)  -  
> 1, n inteiro  positivo, e c a circunferência do disco aberto D(0, 1) . 
> Mostre que:
>
> 1) I_n = Integral_c dz/(P_n(z)) existe para todo n
>
> 2) Dentre os zeros de P_n, existe um, z_n, tal que I_n pode ser expresso em 
> forma fechada como função de n e de z_n.
>
> 3) lim n --> oo I_n = 0
>
> Abraços
>
> Artur
>
>
>
> --
> Esta mensagem foi verificada pelo sistema de antivírus e
>  acredita-se estar livre de perigo.
>
>
> =
> Instruções para entrar na lista, sair da lista e usar a lista em
> http://www.mat.puc-rio.br/~obmlistas/obm-l.html
> =============



-- 
Bernardo Freitas Paulo da Costa

-- 
Esta mensagem foi verificada pelo sistema de antiv�rus e
 acredita-se estar livre de perigo.


=
Instru��es para entrar na lista, sair da lista e usar a lista em
http://www.mat.puc-rio.br/~obmlistas/obm-l.html
=


Re: [obm-l] Problema da olimpiada hungara.

2017-05-17 Por tôpico Bernardo Freitas Paulo da Costa
É bem mais fácil.  "Monte" o produto N*N como na escola.  Vai ficar um
monte de "1" em cada linha e coluna.  A 73ª coluna tem 73 "uns".
Agora, é só ver qual foi o "vai-um" da coluna anterior.  E para isso
tem que ver a anterior da anterior, mas (dica) não precisa ir muito
longe.

Abraços,
-- 
Bernardo Freitas Paulo da Costa

2017-05-16 22:33 GMT-03:00 Anderson Torres <torres.anderson...@gmail.com>:
> N=99...9/9 = (10^2012-1)/9
>
> 9N = 10^2012-1
> 81N^2= 10^4024-2*10^2012+1
>
> Agora tenta aplicar módulo 10^74:
>
> 81N^2= 10^4024-2*10^2012+1
>
> 81N^2=1 (mod 10^74)
>
> Agora teria que achar o "inverso" de 81 módulo 10^74, mas não parece
> fácil de cara.
>
> Outra forma seria usar alguma indução. Pelo que vi no Python, o número
> é bonitinho:
>
> 987654320987654320987654320987654320987654320987654320987654320987654320987654320987654320987654320987654320987654320987654320987654320987654320987654320987654320987654320987654320987654320987654320987654320987654320987654320987654320987654320987654320987654320987654320987654320987654320987654320987654320987654320987654320987654320987654320987654320987654320987654320987654320987654320987654320987654320987654320987654320987654320987654320987654320987654320987654320987654320987654320987654320987654320987654320987654320987654320987654320987654320987654320987654320987654320987654320987654320987654320987654320987654320987654320987654320987654320987654320987654320987654320987654320987654320987654320987654320987654320987654320987654320987654320987654320987654320987654320987654320987654320987654320987654320987654320987654320987654320987654320987654320987654320987654320987654320987654320987654320987654320987654320987654320987654320987654320987654320987654320987654320987654320987654!
 32!
>  
> 0987654320987654320987654320987654320987654320987654320987654320987654320987654320987654320987654320987654320987654320987654320987654320987654320987654321L
>
>
>
>
>
>
> Em 16 de maio de 2017 16:38, Mauricio de Araujo
> <mauricio.de.ara...@gmail.com> escreveu:
>> Dado o numero N = 1...11 formado por 2012 algarismos iguais a 1, qual o
>> algarismo que ocupa a 73a. posição a partir do algarismo das unidades do
>> numero N^2?
>> --
>> Abraços,
>> Mauricio de Araujo
>> [oɾnɐɹɐ ǝp oıɔıɹnɐɯ]
>>

-- 
Esta mensagem foi verificada pelo sistema de antiv�rus e
 acredita-se estar livre de perigo.


=
Instru��es para entrar na lista, sair da lista e usar a lista em
http://www.mat.puc-rio.br/~obmlistas/obm-l.html
=


[obm-l] Re: [obm-l] Probabilidade e números primos

2017-04-07 Por tôpico Bernardo Freitas Paulo da Costa
2017-04-07 21:53 GMT-03:00 Israel Meireles Chrisostomo
<israelmchrisost...@gmail.com>:
> Olá pessoal, eu gostaria de saber como provar que a probabilidade de p²
> dividir um número n é igual a 1/p²(onde p é um número primo).

Probabilidade é sempre mais difícil quando você tem que adivinhar
alguma coisa.  Nesta questão, qual é a distribuição de n?  Não pode
ser uniforme (que é a que a gente "chuta" quando o enunciado é claro
como lama, imaginando que o cara que fez a pergunta é preguiçoso mas
não mal-intencionado), porque não faz sentido uniforme no conjunto N.
Sem isso, eu posso até tentar resolver o "problema inverso" de achar a
distribuição de probabilidades em N tal que P[ n é divisível por p² ]
= 1/p² para todo primo p, mas agora está tarde demais ;-)

Abraços,
-- 
Bernardo Freitas Paulo da Costa

-- 
Esta mensagem foi verificada pelo sistema de antiv�rus e
 acredita-se estar livre de perigo.


=
Instru��es para entrar na lista, sair da lista e usar a lista em
http://www.mat.puc-rio.br/~obmlistas/obm-l.html
=


[obm-l] Re: [obm-l] Equação diferencial

2017-03-06 Por tôpico Bernardo Freitas Paulo da Costa
2017-03-06 9:34 GMT-03:00 Rogério Possi Júnior <roposs...@hotmail.com>:
> Bom dia.
>
>
> Seja a equação y^(4)+a_3*y^(3)+a_2*y^(2)+a_1*y^(1)+a_0*y=0 (aqui y^(n)
> representa a derivada de ordem "n" de y em relação a t). Se
> y(t)=5*t*e^(5*t)+e^(t)*sen(t) é sua solução, determine a_0.
>
> Uma saída (na força) consiste em aplicar a solução na equação dada ...
> caindo em um sistema 4X4 ...
>
> Mas acho que deve ter outra forma mais elegante ... alguém sabe como
> fazê-lo?

Olhe para as raízes do polinômio característico correspondente à
solução.  Se eu não errei as contas, dá a_0 = 50 (e a_2 = 47).

Abraços,
-- 
Bernardo Freitas Paulo da Costa

-- 
Esta mensagem foi verificada pelo sistema de antiv�rus e
 acredita-se estar livre de perigo.


=
Instru��es para entrar na lista, sair da lista e usar a lista em
http://www.mat.puc-rio.br/~obmlistas/obm-l.html
=


[obm-l] Re: [obm-l] Conguência

2017-02-12 Por tôpico Bernardo Freitas Paulo da Costa
2017-02-12 21:55 GMT-02:00 marcone augusto araújo borges
<marconeborge...@hotmail.com>:
> Mostre que 111...11(81 uns) é múltiplo de 81

Vou fazer (na marra) com 3 uns.  Você adapta para 81 = 3^4 uns, é igual:

1) Note que 111 = 999/9 = (10^3 - 1)/9
2) Para mostrar que 111 é divisível por 3, "basta" mostrar que (10^3 -
1) é divisível por 27.
3) 10^2 = 100 = 3*30 + 10 == 3*3 + 10 = 19 mod 27
4) 10^3 == 10*19 = 190 = 10 + 180 = 10 + 6*30 == 10 + 6*3 = 28 == 1 mod 27

(Sugestão para calcular 10^81: calcule 10^5, depois eleve ao quadrado
até chegar em 10^80, e no final multiplique por 10)


Se você quiser provar o caso geral (3^n "uns" é divisível por 3^n)
você na verdade vai ter que provar que 10^(3^n) - 1 é *exatamente*
divisível por 3^(n+2), por indução.

Abraços,
-- 
Bernardo Freitas Paulo da Costa

-- 
Esta mensagem foi verificada pelo sistema de antiv�rus e
 acredita-se estar livre de perigo.


=
Instru��es para entrar na lista, sair da lista e usar a lista em
http://www.mat.puc-rio.br/~obmlistas/obm-l.html
=


[obm-l] Re: [obm-l] Re: [obm-l] qual destes é o maior?

2017-02-10 Por tôpico Bernardo Freitas Paulo da Costa
2017-02-10 11:37 GMT-02:00 Rodrigo Costa <costa@gmail.com>:
>
> Eu tenho uma sugestão... pode não ser muito elegante mas daria para usar a 
> aproximação de stirling para os termos em fatorial.

Muita gente consideraria isso "roubar".  Eu sempre acho muito
artificial um problema de fatoriais onde você não deixa usar Stirling:
a maior parte dos problemas NASCEU de alguém que usou Stirling
primeiro, e depois tentou descobrir uma demonstração (já sabendo o
resultado certo!) "elementar" (mas obscura).

> E mostrar que o limite de 2^(n!)/(2^n)! -> inf e (2^n)!/2^(n!) -> 0 para 
> n->inf com a aproximação.
>
> i.e. 2^(n!) > 2^(n)! para n>=5...

Gostei da figura, e da técnica.  Aliás, isso é bastante geral: se você
não sabe qual é o maior número, manda um log que as coisas devem ficar
claras.  Ou dois logs.  Ou mais...  Aqui, dois bastam:

log[ 2^(n!) ] = n! * log(2)
log[ (2^n)! ] ~ 2^n * log(2^n) = 2^n * n * log(2) (tem um "termo de
erro" que é - 2^n, mas isso é bem menor do que o termo anterior, e tem
outros termos de erro também.  A graça disso é fazer BEM rápido)

Aqui, já dá para ver como apareceu o "passo mágico" da resposta do
Gabriel: para comparar estes logs (aproximados), você tem que comparar
n! e 2^n * n.  Se você já souber fazer isso, ótimo.  Se não, venha o
segundo log:

log[ n! ] ~ n * log(n)
log[ 2^n * n ] = n*log(2) + log(n)

E daí está na cara que n*log(n) é muito maior do que n*log(2) (que por
sua vez é MUITO maior do que log(n)).

Abraços,
--
Bernardo Freitas Paulo da Costa

-- 
Esta mensagem foi verificada pelo sistema de antiv�rus e
 acredita-se estar livre de perigo.


=
Instru��es para entrar na lista, sair da lista e usar a lista em
http://www.mat.puc-rio.br/~obmlistas/obm-l.html
=


Re: [obm-l] Integral e Derivada

2017-02-08 Por tôpico Bernardo Freitas Paulo da Costa
2017-02-08 16:26 GMT-02:00 Anselmo Alves de Sousa <starterm...@gmail.com>:
> Solicito auxílio pra resolver:
>
> 1. \int_1^{\infty} \sqrt{\frac{1-\cos(x)}{x^3}} dx

Ela é claramente finita.
O *quanto* ela vale, acho que só numericamente; acho que nem com
resíduos sai.  E como o integrando nem é diferenciável, vai dar
trabalho...

> 2. obter a derivada de f(x) = \int_{x}^{\sqrt{x}}\frac{\exp(xy^2)}{y} dy

Tem que ter cuidado com a derivada, mas não é difícil.  Você vai ficar
com três termos: um derivando no limite inferior, outro no limite
superior (! cuidado: regra da cadeia para a sqrt) e depois a derivada
dentro da integral.  A "parte boa" é que a derivada dentro da integral
fará aparecer um y^2 que permite efetuar analiticamente a integral
depois da mudança t = y^2, dt = 2ydy. (antes não daria certo)

Abraços,
-- 
Bernardo Freitas Paulo da Costa

-- 
Esta mensagem foi verificada pelo sistema de antiv�rus e
 acredita-se estar livre de perigo.


=
Instru��es para entrar na lista, sair da lista e usar a lista em
http://www.mat.puc-rio.br/~obmlistas/obm-l.html
=


Re: [obm-l] Decrescimento

2017-01-12 Por tôpico Bernardo Freitas Paulo da Costa
2017-01-11 23:38 GMT-02:00 Israel Meireles Chrisostomo
<israelmchrisost...@gmail.com>:
>
> Olá pessoal gostaria de saber se a função é decrescente

Não, pois em infinitos pontos ela tende a infinito.
-- 
Bernardo Freitas Paulo da Costa

-- 
Esta mensagem foi verificada pelo sistema de antiv�rus e
 acredita-se estar livre de perigo.


=
Instru��es para entrar na lista, sair da lista e usar a lista em
http://www.mat.puc-rio.br/~obmlistas/obm-l.html
=


[obm-l] Re: [obm-l] Re: [obm-l] Polinômio irredutível em Z

2016-11-23 Por tôpico Bernardo Freitas Paulo da Costa
2016-11-23 14:21 GMT-02:00 Anderson Torres <torres.anderson...@gmail.com>:
> Isso não me parece verdadeiro - (2x-1)^1000 é um contraexemplo.

Acho que tem uma hipótese implícita de que todas as raízes são distintas.

Abraços,
-- 
Bernardo Freitas Paulo da Costa

> Em 13 de novembro de 2016 14:20, Adrian Alexander Delgado
> <adrian.alexander4...@gmail.com> escreveu:
>> É sobre esse problema:
>> (Irã 2007) Existe uma sequência de inteiros a_0, a_1, a_2, ... tais que
>> (a_i,a_j)=1 para i diferente de j e para todo n inteiro positivo a_0 + a_1 x
>> +... +a_n x^n é irredutível em Z[x]?
>>
>> No fórum AoPS, vi que a solução usa o fato de que
>> Se toda raiz complexa α de f satisfaz |α|<1, então f é irredutível em Z
>>
>> Tentei procura uma demonstração disso na internet e não encontrei.
>> Alguém sabe como demonstrar isso?
>>
>> Link da solução:
>> http://artofproblemsolving.com/community/c6h149740p847418
>>

-- 
Esta mensagem foi verificada pelo sistema de antiv�rus e
 acredita-se estar livre de perigo.


=
Instru��es para entrar na lista, sair da lista e usar a lista em
http://www.mat.puc-rio.br/~obmlistas/obm-l.html
=


[obm-l] Re: [obm-l] Re: Números complexos

2016-08-21 Por tôpico Bernardo Freitas Paulo da Costa
2016-08-20 20:33 GMT-03:00 Israel Meireles Chrisostomo
<israelmchrisost...@gmail.com>:
> Ah todos os valores reais de x

Deixa eu escrever o enunciado que eu acho que você quis

"Encontre todos os pares (x,n) tais que (x+i)^{4n} seja um número real".

Acertei? Se for isso, para cada "n" haverá algumas soluções. E sem
pensar muito, eu acho que (exceto n=1) há 2n soluções, com "n" fixo.

A primeira parte de fazer uma pergunta é escrevê-la com clareza: pense
um pouco mais antes de apertar "send", ajuda a lista a te ajudar (ou
então seja claro ao dizer que não sabe muito bem como formular a
pergunta e peça ajuda para isso também!)

> Em 20 de agosto de 2016 20:02, Israel Meireles Chrisostomo
> <israelmchrisost...@gmail.com> escreveu:
>>
>> Olá pessoal, gostaria de saber se é possível resolver a equação:
>>
>> (x+i)^{4n}=Re(z)
>>
>> onde z é um número complexo qualquer e Re(z) denota a parte real de z.Isto
>> é, em outras palavras gostaria de saber todos os valores de x para os quais
>> (x+i)^{4n} é real .De cara, dá para ver que x=0 é um desses valores, mas
>> como provo que esse é o único(ou não)...

Abraços,
-- 
Bernardo Freitas Paulo da Costa

-- 
Esta mensagem foi verificada pelo sistema de antiv�rus e
 acredita-se estar livre de perigo.


=
Instru��es para entrar na lista, sair da lista e usar a lista em
http://www.mat.puc-rio.br/~obmlistas/obm-l.html
=


[obm-l] Re: [obm-l] [obm-l] Polinômios

2016-08-02 Por tôpico Bernardo Freitas Paulo da Costa
2016-08-02 18:29 GMT-03:00 Daniel Rocha <daniel.rocha@gmail.com>:
> Alguém poderia, por favor, solucionar o problema abaixo:
>
> O resto da divisão de um polinômio P(x) por (2x - 1) é 4; deste modo, o
> resto da divisão de (x^2 - x)*P(x) por (2x - 1) é:

Relacione o resto da divisão com o valor do polinômio em algum "x" "esperto".

Abraços,
-- 
Bernardo Freitas Paulo da Costa

-- 
Esta mensagem foi verificada pelo sistema de antiv�rus e
 acredita-se estar livre de perigo.


=
Instru��es para entrar na lista, sair da lista e usar a lista em
http://www.mat.puc-rio.br/~obmlistas/obm-l.html
=


[obm-l] Re: [obm-l] Re: [obm-l] Projeção Ortogonal

2016-05-25 Por tôpico Bernardo Freitas Paulo da Costa
2016-05-24 22:34 GMT-03:00 Kelvin Anjos <kelvinan...@gmail.com>:
> A projeção ortogonal de uma parábola sempre será congruente à sua diretriz,

Essa frase eu entendi, mas gostaria de uma demonstração.

> isto é, a projeção será a reta coincidente ao eixo paralelo à sua diretriz
> em coordenadas cartesianas.

A projeção não devia ser uma reta ortogonal à direção de projeção?
(Que não precisa ter relação com a diretriz?) Sei lá, imagine que você
está projetando na reta x=y.



Ainda acho que falta deixar o enunciado mais claro...

Abraços,
-- 
Bernardo Freitas Paulo da Costa

-- 
Esta mensagem foi verificada pelo sistema de antiv�rus e
 acredita-se estar livre de perigo.


=
Instru��es para entrar na lista, sair da lista e usar a lista em
http://www.mat.puc-rio.br/~obmlistas/obm-l.html
=


[obm-l] Re: [obm-l] Projeção Ortogonal

2016-05-24 Por tôpico Bernardo Freitas Paulo da Costa
2016-05-24 19:41 GMT-03:00 Daniel Rocha <daniel.rocha@gmail.com>:
> Alguém poderia, por favor, esclarecer a seguinte dúvida:
>
> A Projeção Ortogonal de uma Parábola pode resultar em um Segmento de Reta???

O que é um "segmento de reta"? Pode ser (-oo, +oo)? Pode ser (0, +
oo)? Ou tem que ser um segmento LIMITADO da reta?

O que é uma projeção ortogonal? Vale de R^3 em R? Ou é só de R^2 num
subespaço afim / linear?

> Sim ou Não.


Abraços,
-- 
Bernardo Freitas Paulo da Costa

-- 
Esta mensagem foi verificada pelo sistema de antiv�rus e
 acredita-se estar livre de perigo.


=
Instru��es para entrar na lista, sair da lista e usar a lista em
http://www.mat.puc-rio.br/~obmlistas/obm-l.html
=


[obm-l] Re: [obm-l] Aritmética

2016-03-30 Por tôpico Bernardo Freitas Paulo da Costa
2016-03-30 16:55 GMT-03:00 Pedro Júnior <pedromatematic...@gmail.com>:
> Qual o maior resto possível da divisão de um número de dois algarismos pela
> soma de seus algarismos?
15.

> Caso saibam de alguma fórmula ou teoria gostaria do link ou referência.

Eu sei do meu computador. Segue uma lista dos pares (n, resto), cada
vez que o resto fica maior do que os restos anteriores

11 1
14 4
19 9
49 10
68 12
79 15
299 19
689 22
799 24
3889 25
4898 26
5599 27
6698 28
7996 29
8798 30
9599 31
16999 33
18899 34
39899 37
4 39
89789 40
89998 42
98999 43

Curiosamente, começa com os quadrados (1,4,9), depois tem uma
"carreirinha" de 24-31, pula 32 = 2^5, 35, 36, 38 que são compostos,
mas logo pula 41 que é primo. (já tinha pulado outros, mas você
poderia imaginar que era um "defeito" de ter "muito poucos dígitos")

Abraços,
-- 
Bernardo Freitas Paulo da Costa

-- 
Esta mensagem foi verificada pelo sistema de antiv�rus e
 acredita-se estar livre de perigo.


=
Instru��es para entrar na lista, sair da lista e usar a lista em
http://www.mat.puc-rio.br/~obmlistas/obm-l.html
=


[obm-l] Re: [obm-l] Existência de Função

2016-03-22 Por tôpico Bernardo Freitas Paulo da Costa
2016-03-22 5:11 GMT-03:00 Pedro Júnior <pedromatematic...@gmail.com>:
> Se f: X --> Y é injetiva e g: N --> Y é bijetiva, mostre que existe h: N -->
> X bijetiva.
>
> obs.: N:= naturais

Isso é falso. Tome X = {1}, Y = N. f(1) = 1 é injetiva (toda função de
um conjunto com um único elemento é injetiva!). g é a identidade, que
é obviamente bijetiva. Não existe h : N -> X porque X não é infinito.

Se você pedir que X seja infinito, então é verdade, porque "N é o
menor infinito", e a hipótese é equivalente a existir f2 : X -> N
injetiva.

Abraços,
-- 
Bernardo Freitas Paulo da Costa

-- 
Esta mensagem foi verificada pelo sistema de antiv�rus e
 acredita-se estar livre de perigo.


=
Instru��es para entrar na lista, sair da lista e usar a lista em
http://www.mat.puc-rio.br/~obmlistas/obm-l.html
=


Re: [obm-l] Re: Mostrar que Soma (k = 1, n) 1/P'(x_k) = 0

2016-02-19 Por tôpico Bernardo Freitas Paulo da Costa
Olá,

a cara desta expressão (soma nas raízes de um polinômio) me faz pensar
em integral de Cauchy / resíduos. Um lado você consegue com a integral
olhando para dentro de um círculo bem grande (contendo todas as
raízes) e a outra "no lado de fora do círculo". Não estou com tempo de
pensar qual seria a função, mas acho que é por aí. Talvez a noite
traga inspiração?

Abraços,
-- 
Bernardo Freitas Paulo da Costa

2016-02-19 18:19 GMT-02:00 Luís <qed_te...@hotmail.com>:
> Sauda,c~oes,
>
> Parece que não chegou. Mando novamente.
>
> Luís
>
> 
> De: Luís <qed_te...@hotmail.com>
> Enviado: sexta-feira, 19 de fevereiro de 2016 14:35
> Para: obm-l@mat.puc-rio.br
> Assunto: Re: Mostrar que Soma (k = 1, n) 1/P'(x_k) = 0
>
> Sauda,c~oes, oi Amanda,
>
> Apesar de não conseguir uma solução, gostei deste problema.
>
> Para n=2 e n=3 podemos ver que isso é verdade.
>
> P[2](x)=(x - x_1) (x - x_2)
> P[2]'(x) = (x - x_2) +  (x - x_1)
> P[2]'(x_1) = (x_1 - x_2)
> P[2]'(x_2) = (x_2 - x_1)
>
> 1/P[2]'(x_1) + 1/P[2]'(x_2) = 1/(x_1 - x_2) + 1/ (x_2 - x_1) = 0
>
> P[3](x) = (x - x_1) (x - x_2) (x - x_3)
> P[3]'(x) =  (x - x_2) (x - x_3) +  (x - x_1)  (x - x_3) +  (x - x_1) (x - x_2)
> P[3]'(x_1) = (x_1 - x_2) (x_1 - x_3)
> P[3]'(x_2) = (x_2 - x_1) (x_2 - x_3)
> P[3]'(x_3) = (x_3 - x_1) (x_3 - x_2)
>
> Soma (k = 1,3) 1/P[3]'(x_k) = 0
>
> Mostrar assim para n >= 4 começa a ficar longo. Pensei então em indução.
>
> P[n+1](x) = (x - x_(n+1)) Q[n](x)
>
> P[n+1]'(x) = Q[n](x) + (x - x_(n+1)) Q[n]'(x)
>
> Soma (k = 1,n) 1/Q[n]'(x)  = 0 (hipótese da indução)
>
> Soma (k = 1,n+1) 1/P[n+1]'(x_k) = Soma (k = 1,n) 1 / [ (x_k - x_(n+1)) 
> Q[n]'(x_k) ]  + 1 / Q[n](x_(n+1))
>
> Parei aqui. Será que é possível manipular a expressão e chegar ao resultado ?
>
> Luís
>
>
> 
> De: owner-ob...@mat.puc-rio.br <owner-ob...@mat.puc-rio.br> em nome de Merryl 
> <sc...@hotmail.com>
> Enviado: terça-feira, 16 de fevereiro de 2016 23:18
> Para: obm-l@mat.puc-rio.br
> Assunto: [obm-l] Mostrar que Soma (k = 1, n) 1/P'(x_k) = 0
>
> Tentando mostrar isto, cheguei a uma expressão extremamente complicada. Podem 
> ajudar?
>
> Seja P um polinômio de grau n >= 2 tal que suas n raízes x_1, ... x_n sejam 
> distintas duas a duas. Mostre que
>
> Soma (k = 1, n) 1/P'(x_k) = 0
>
> Obrigada
>
> Amanda

-- 
Esta mensagem foi verificada pelo sistema de antiv�rus e
 acredita-se estar livre de perigo.


=
Instru��es para entrar na lista, sair da lista e usar a lista em
http://www.mat.puc-rio.br/~obmlistas/obm-l.html
=


[obm-l] Re: [obm-l] Números inteiros

2016-01-24 Por tôpico Bernardo Freitas Paulo da Costa
2016-01-24 22:30 GMT-02:00 marcone augusto araújo borges
<marconeborge...@hotmail.com>:
> Determinar o número de soluções inteiras da equação a + b + c + d = 27
> onde cada variável toma valores entre 3 e 8

Faça a = A + 3, idem para B, C, D. Isso dá

A+B+C+D = 27 - 4*3 = 15, onde A,B,C,D estão entre 0 e 5. Daqui em
diante, o argumento de separar pedras (os 15 totais) com pauzinhos
(para escolher quantas vão para A,B,C ou D) mata.

Abraços,
-- 
Bernardo Freitas Paulo da Costa

=
Instru��es para entrar na lista, sair da lista e usar a lista em
http://www.mat.puc-rio.br/~obmlistas/obm-l.html
=


[obm-l] Re: [obm-l] Cálculo - Integral dupla

2016-01-10 Por tôpico Bernardo Freitas Paulo da Costa
2016-01-10 22:11 GMT-02:00 Roger <roger@gmail.com>:
> Essa é uma questão pra eng. da Petrobrás, do concurso de 2012. Mas faz uns
> dois dias que não acho a solução.
>
> integral dupla
>
> int (0 a 1) int (y a 1) 2*e^(-x^2) dxdy

Como e^(-x^2) não tem primitiva analítica, provavelmente você tem que
mudar a ordem de integração para conseguir fazer alguma coisa. Como é
uma questão da Petrobrás, acredito que basta fazer isso e vai sair.

> a resposta oficial é 1 - 1/e.

Você pode tentar verificar no Wolfram Alfa. Não sei se ele vai ser
esperto o bastante para fazer a mudança da ordem de integrais, mas uma
vez que você tenha feito dxdy virar dydx e mudado os limites, o
wolfram deve dar a resposta pra você.


-- 
Bernardo Freitas Paulo da Costa

=
Instru��es para entrar na lista, sair da lista e usar a lista em
http://www.mat.puc-rio.br/~obmlistas/obm-l.html
=


[obm-l] Re: [obm-l] Função Convexidade

2015-12-07 Por tôpico Bernardo Freitas Paulo da Costa
2015-12-07 9:42 GMT-02:00 Israel Meireles Chrisostomo
<israelmchrisost...@gmail.com>:
> Olá rapazes, será que alguém poderia confirmar para mim que a função √senx é
> côncova no intervalo (0,pi/2)?

http://www.wolframalpha.com/input/?i=d%C2%B2%2Fdx%C2%B2%28sqrt%28sin%28x%29%29
-- 
Bernardo Freitas Paulo da Costa

-- 
Esta mensagem foi verificada pelo sistema de antiv�rus e
 acredita-se estar livre de perigo.


=
Instru��es para entrar na lista, sair da lista e usar a lista em
http://www.mat.puc-rio.br/~obmlistas/obm-l.html
=


Re: [obm-l] Probabilidades com Dados

2015-12-03 Por tôpico Bernardo Freitas Paulo da Costa
2015-12-03 18:43 GMT-02:00 Marcelo Salhab Brogliato <msbro...@gmail.com>:
> Acho que sai usando funções geradoras.
Sempre sai, principalmente com um computador para fazer as contas ;-)

> A resposta seria o coeficiente de x^21 da expansão (1/6x + 1/6x^2 + 1/6x^3 +
> 1/6x^4 + 1/6x^5 + 1/6x^6)^6 = (x + x^2 + x^3 + x^4 + x^5 + x^6)^6 / 6^6.
>
> Vejo alguns possíveis caminhos:
>
> 1) Veja que: x + x^2 + x^3 + x^4 + x^5 + x^6 = x(x^6 - 1) / (x-1).
> Agora temos que dividir (x^6-1)^6 por (x-1)^6 e pegar o coeficiente de
> x^(21-6) = x^15.
> Usando Briot-Rufini, acho que dá para pegar esse coeficiente sem ter que
> abrir tudo, mas não tenho certeza.

Tem um jeito mais fácil depois que você viu isso. Temos que achar o
coeficiente de x^21 em [ x(x^6 - 1)/(x - 1) ]^6, que é, como você
disse, o coeficiente de x^15 em [ (x^6 - 1)/(x - 1) ]^6. Que é o
coeficiente de x^15 em (x^6 - 1)^6 / (x - 1)^6 = (1 - x^6)^6 / (1 -
x)^6. O numerador tem apenas três termos que contribuem, a saber:
1, -6x^6, binom(6,2)x^12

Isso quer dizer que temos que achar os coeficientes de x^15, x^9 e x^3
na expansão de (1 - x)^{-6}. Ora, pelo teorema binomial, é
simplesmente:
binom(-6,15)(-1)^15, binom(-6,9)(-1)^9, binom(-6,3)(-1)^3. Agora, note
que (-6,k) = (-6)(-6-1) ... (-6-k+1)/k! = (-1)^k * binom(6+k-1,k) =
(-1)^k binom(5+k,k) = (-1)^k binom(5+k,5).

Pronto, agora é só multiplicar e somar:
binom(20,5) - 6*binom(14,5) + binom(6,2)*binom(8,5) = 15504 - 6*2002 +
15*56 = 4332.

Ah, sim, não esqueça de dividir por 6^6.
-- 
Bernardo Freitas Paulo da Costa

-- 
Esta mensagem foi verificada pelo sistema de antiv�rus e
 acredita-se estar livre de perigo.


=
Instru��es para entrar na lista, sair da lista e usar a lista em
http://www.mat.puc-rio.br/~obmlistas/obm-l.html
=


Re: [obm-l] Primo?

2015-11-24 Por tôpico Bernardo Freitas Paulo da Costa
2015-11-24 7:32 GMT-02:00 marcone augusto araújo borges
<marconeborge...@hotmail.com>:
> Mostre que 2^83 - 1 não é primo

Seja p um primo que divide 2^83 - 1. Seja "x" a ordem de 2 mod p. Por
Fermat, sabemos que x divide (p-1). Do enunciado, x também divide 83.
Ponha (p-1) = kx, e vá aumentando k até achar um primo que, realmente,
divida 2^83 - 1...

Abraços,
-- 
Bernardo Freitas Paulo da Costa

-- 
Esta mensagem foi verificada pelo sistema de antiv�rus e
 acredita-se estar livre de perigo.


=
Instru��es para entrar na lista, sair da lista e usar a lista em
http://www.mat.puc-rio.br/~obmlistas/obm-l.html
=


Re: [obm-l] Derivada de um valor esperado

2015-11-03 Por tôpico Bernardo Freitas Paulo da Costa
2015-11-02 17:26 GMT-02:00 Amanda Merryl <sc...@hotmail.com>:
> Uma usina hidrelétrica deve atender uma carga de potência conhecida s. A 
> potência P disponível na hidrelétrica é uma variável aleatória distribuida em 
> [0, Pmax] segundo uma função distribuição de probabilidade contínua. O 
> déficit de potência D é definido por
>
> D = max(s - P, 0)
>
> e a probabilidade de perda de carga para a carga s é definida como l(s) = 
> Prob(D > 0).
>
> Mostre que d/E[D]/ds = l(s), sendo E[D] o valor esperado de D.

O que você sabe de probabilidade contínua? Por exemplo, você já viu a
formulação integral da esperança?
-- 
Bernardo Freitas Paulo da Costa

-- 
Esta mensagem foi verificada pelo sistema de antiv�rus e
 acredita-se estar livre de perigo.


=
Instru��es para entrar na lista, sair da lista e usar a lista em
http://www.mat.puc-rio.br/~obmlistas/obm-l.html
=


[obm-l] Re: [obm-l] números especiais OMERJ 2015

2015-10-15 Por tôpico Bernardo Freitas Paulo da Costa
2015-10-15 21:43 GMT-03:00 Bernardo Freitas Paulo da Costa
<bernardo...@gmail.com>:
> 2015-10-15 16:42 GMT-03:00 Luís <qed_te...@hotmail.com>:
>> Sauda,c~oes,
>>
>> Um número é dito especial se ele tem dois ou mais algarismos
>> e é múltiplo da soma dos seus algarismos. Por exemplo, 12 é
>> especial pois é múltiplo de 1 + 2 = 3.
>>
>> a) encontre três números especiais consecutivos;
>
> Não pensei em nada muito especial, mas se x = ab com dois dígitos
> ímpares, a soma é par.
> Logo, para 1b, 3b, 5b, 7b e 9b não precisa testar se os números são
> especiais ou não, pois é impossível haver três consecutivos, já que a
> cada dois um não é. Logo restam os números da forma 2b, 4b, 6b e 8b
> (com dois dígitos). Daí em diante um pouco de força bruta acha três
> consecutivos.
>
>> b) encontre quatro números especiais consecutivos.
>
> Ainda não achei estes. Acredito que tenha que usar divisibilidade por
> 3 e 4; eu não usei por 3 no caso anterior porque acabou sendo mais
> fácil a força bruta mesmo.

Achei. Divisibilidade mesmo. Vou dar uma dica: escreva o primeiro
número dos quatro na forma (10x + b), e introduza s = soma dos dígitos
de x. Há alguns casos a tratar, mas "aposte na sorte" e nos casos que
"têm mais": suponha que não ocorre vai-um. Com isso, escreva as
condições de divisibilidade, e mostre que (10x - s) tem que ser bem
especial.

Problema muito bonito!

Abraços
-- 
Bernardo Freitas Paulo da Costa

-- 
Esta mensagem foi verificada pelo sistema de antiv�rus e
 acredita-se estar livre de perigo.


=
Instru��es para entrar na lista, sair da lista e usar a lista em
http://www.mat.puc-rio.br/~obmlistas/obm-l.html
=


[obm-l] Re: [obm-l] números especiais OMERJ 2015

2015-10-15 Por tôpico Bernardo Freitas Paulo da Costa
2015-10-15 16:42 GMT-03:00 Luís <qed_te...@hotmail.com>:
> Sauda,c~oes,
>
> Um número é dito especial se ele tem dois ou mais algarismos
> e é múltiplo da soma dos seus algarismos. Por exemplo, 12 é
> especial pois é múltiplo de 1 + 2 = 3.
>
> a) encontre três números especiais consecutivos;

Não pensei em nada muito especial, mas se x = ab com dois dígitos
ímpares, a soma é par.
Logo, para 1b, 3b, 5b, 7b e 9b não precisa testar se os números são
especiais ou não, pois é impossível haver três consecutivos, já que a
cada dois um não é. Logo restam os números da forma 2b, 4b, 6b e 8b
(com dois dígitos). Daí em diante um pouco de força bruta acha três
consecutivos.

> b) encontre quatro números especiais consecutivos.

Ainda não achei estes. Acredito que tenha que usar divisibilidade por
3 e 4; eu não usei por 3 no caso anterior porque acabou sendo mais
fácil a força bruta mesmo.
-- 
Bernardo Freitas Paulo da Costa

-- 
Esta mensagem foi verificada pelo sistema de antiv�rus e
 acredita-se estar livre de perigo.


=
Instru��es para entrar na lista, sair da lista e usar a lista em
http://www.mat.puc-rio.br/~obmlistas/obm-l.html
=


Re: [obm-l] Problema 6 da OBM de 2002

2015-10-12 Por tôpico Bernardo Freitas Paulo da Costa
2015-10-12 0:31 GMT-03:00 Gabriel Tostes <gtos...@icloud.com>:
> Mostre que não podemos formar mais que 4096 sequências binárias de tamanho 24 
> tal que quaisquer 2 diferem em ao menos 8 posições.
> Não consegui entender a resolução na Eureka. Alguém pode resolvê-lo?

Eu não sei se conheço alguma solução além da do Fábio (imagio que seja
esta a da Eureka). Mas acredito que pode ajudar a entender o problema
diminuindo os números, e tentando ser mais ambicioso: tente descobrir
a maior quantidade de sequências binárias de tamanho 4 tais que duas
quaisquer diferem em ao menos 2 posições. Vou tentar começar o
raciocínio: Suponha, sem perda de generalidade, que uma das sequências
é a . Então, você não pode ter nenhuma sequência com apenas um
"1", certo? Agora, pense em quantas sequências com dois "1" podem
haver. Ao todo, há 6, mas você não pode escolher todas elas, certo?

Para completar, ainda falta a idéia das "regiões de influência" (cada
sequência escolhida "domina" algumas sequências, as que estão mais
próximas dela do que de qualquer outra sequência). Para visualisar
isso, pense que, em vez de 4/2, o problema é sobre tam=5 / diferença
>= 3. Faça um "ponto" para cada sequência (dá 32, dá trabalho mas é
factível) e ligue as que diferem de apenas uma posição. Daí, comece
marcando uma (tipo a 0) e depois vá escolhendo como puder. Isso
deve deixar claro para você que cada sequência tem uma região de
influência de tamanho 1.

Abraços,
-- 
Bernardo Freitas Paulo da Costa

-- 
Esta mensagem foi verificada pelo sistema de antiv�rus e
 acredita-se estar livre de perigo.


=
Instru��es para entrar na lista, sair da lista e usar a lista em
http://www.mat.puc-rio.br/~obmlistas/obm-l.html
=


Re: [obm-l] Inteiros

2015-10-06 Por tôpico Bernardo Freitas Paulo da Costa
2015-10-06 20:23 GMT-03:00 marcone augusto araújo borges
<marconeborge...@hotmail.com>:
> Determine todos os pares de inteiros x e y tais que x^2 - 2xy + 125y^2 =
> 2009

Isso dá (x-y)^2 + 124y^2 = 2009. Chame (x-y) de z, fica z^2 + 124y^2 =
2009. Daí:

y^2 < 2009/124 ~ 2000/125 = 16,

então basta testar y = 0, 1, 2, 3 e 4. Para cada um deles, você vê se
tem algum z cujo quadrado seja 2009 - 124y^2, se tiver isso dá duas
soluções em z, e duas soluções em y, o que dá quatro ao todo para
(x,y).


Para não parecer que é mágica, note que toda forma quadrática real em
duas variáveis corresponde a
- elipse (ax^2 + by^2)
- parábola (ax^2)
- hipérbole (ax^2 - by^2)
onde eu botei entre parênteses a forma reduzda após uma mudança de variáveis.

Abraços,
-- 
Bernardo Freitas Paulo da Costa

-- 
Esta mensagem foi verificada pelo sistema de antiv�rus e
 acredita-se estar livre de perigo.


=
Instru��es para entrar na lista, sair da lista e usar a lista em
http://www.mat.puc-rio.br/~obmlistas/obm-l.html
=


[obm-l] Re: [obm-l] Cálculo 2 Serge Lang

2015-09-16 Por tôpico Bernardo Freitas Paulo da Costa
2015-09-16 21:04 GMT-03:00 Israel Meireles Chrisostomo
<israelmchrisost...@gmail.com>:
>
> Alguém sabe onde encontro na net o pdf do livro Cálculo 2 do Serge Lang?

Israel, esta lista é para discutir problemas olímpicos e relacionados,
não o que você pediu. E não sei se o Lang de Cálculo seria uma
referência tão importante e rara assim para justificar a sua mensagem.

-- 
Bernardo Freitas Paulo da Costa

-- 
Esta mensagem foi verificada pelo sistema de antiv�rus e
 acredita-se estar livre de perigo.


=
Instru��es para entrar na lista, sair da lista e usar a lista em
http://www.mat.puc-rio.br/~obmlistas/obm-l.html
=


[obm-l] Re: [obm-l] Derivada de um produto de funções

2015-09-14 Por tôpico Bernardo Freitas Paulo da Costa
2015-09-14 0:48 GMT-03:00 Israel Meireles Chrisostomo
<israelmchrisost...@gmail.com>:
> A fórmula da derivada de um produto de funções vale quando se tem infinitas
> funções?
> Isto é, vale que
> d/dx(f_1(x)f_2(x)f_3(x)...f_n(x)) =
> f '_1(x)f_2(x)f_3(x)...f_n(x)+f_1(x)f'_2(x)f_3(x)...f_n(x) + f_1(x)f_2(x)f 
> '_3(x)...f_n(x)+

Depende. Formalmente, é isso mesmo. Se você quiser estudar mais,
observe que a fórmula fácil corresponde à derivada logarítmica:

d(log f) = f'/f
d(log (f*g)) = f'/f + g'/g
...
d(log (Prod f_i)) = Sum f_i'/f_i

O problema então é garantir que
1) O seu produto infinito realmente faz sentido (ou seja, converge).
Isso depende de as funções ficarem "perto de 1" no infinito
2) A série das derivadas logarítmicas converge

Num curso de análise complexa, em geral você prova as duas usando os
mesmos argumentos, então muitas vezes é "simples assim". Um livro que
faz "isso mesmo" é o do Stein & Shakarshi, se não me engano no
capítulo 5.

Mas lembre-se, muitas coisas que funcionam em complexos só dão certo
graças ao caráter analítico das funções. Se você quiser teoremas
"gerais", em geral, como disse o Artur, as funções reais vão ser bem
"patológicas" e vai dar errado.

Abraços
-- 
Bernardo Freitas Paulo da Costa

-- 
Esta mensagem foi verificada pelo sistema de antiv�rus e
 acredita-se estar livre de perigo.


=
Instru��es para entrar na lista, sair da lista e usar a lista em
http://www.mat.puc-rio.br/~obmlistas/obm-l.html
=


Re: [obm-l] Derivada de e^z

2015-09-10 Por tôpico Bernardo Freitas Paulo da Costa
2015-09-10 21:18 GMT-03:00 Eduardo Henrique <dr.dhe...@outlook.com>:
>
> Ah, z é complexo. Jurava ter escrito isso, desculpe. Sim, pela definição de 
> derivada: lim_{h\rightarrow0}[f(z+h)-f(z)]/h

Só para ser preciso: qual é a sua definição de exp(z) para z complexo?
Qualquer que seja, tem como fazer usando apenas as definições, é só
uma questão de quanto trabalho vai dar. (Eu fiz isso no meu curso
período passado com a definição exp(x + iy) = exp(x)*(cos(y) + i
sin(y)), é cansativo mas "sai")

> De:"Eduardo Henrique" <dr.dhe...@outlook.com>
> Data:18:54 Qui, 10 de Set de PM
> Assunto:[obm-l] Derivada de e^z
>
> Pessoal, to batendo a cabeça aqui faz uns dois dias e não sai. Tem como 
> provar pela definição que a derivada de e^z é e^z?

Abraços,
-- 
Bernardo Freitas Paulo da Costa

-- 
Esta mensagem foi verificada pelo sistema de antiv�rus e
 acredita-se estar livre de perigo.


=
Instru��es para entrar na lista, sair da lista e usar a lista em
http://www.mat.puc-rio.br/~obmlistas/obm-l.html
=


Re: [obm-l] Limites

2015-09-08 Por tôpico Bernardo Freitas Paulo da Costa
2015-09-08 22:24 GMT-03:00 Israel Meireles Chrisostomo
<israelmchrisost...@gmail.com>:
> Já vi uma maneira mais simples é só definir A_n=((2n)!/n!)^{1/n}  e usar que
> (A_n+1)^{n+1}/(A_n)^{n}=A_n(A_n+1/A_n)^{n+1} e observar que lim
> (A_n+1/A_n)^{n+1} =1, essa é uma boa técnica ehehehe

É. Se eu entendi direito, você "substituiu o teste da raiz pelo teste
da razão". Mais explicitamente, se a_n é uma seqüência de números
reais positivos, então se existir o limite a_{n+1} / a_n (quando n ->
infinito), então também existe o limite (a_n)^{1/n} e eles são iguais.
(Acho que você esqueceu de dizer que o lado ESQUERDO da sua equação
tende a 4 quando n -> infinito)

> Em 8 de setembro de 2015 21:42, Israel Meireles Chrisostomo
> <israelmchrisost...@gmail.com> escreveu:
>>
>> Acho que pensei numa forma mais simples

Tem uma outra forma "bem simples". Enfim, super-mágica, mas como eu
estou usando números de Catalan de montão, esses truques acabam
aparecendo. Seja C_n = binom(2n,n). Considere a função 1/raiz(1 - z).
Pelo binômio de Newton, a série de potências dela é

1/raiz(1 - z) = soma (-1)^n (2n+1)!/(4^n n! n!) z^n = soma (-1)^n
(2n+1) C_n/4^n z^n = soma a_n z^n

Como o raio de convergência desta função é 1, sabemos (pelo critério
de Hadamard) que o limite |a_n|^{1/n} é igual a 1. Daí, basta ver que
tem um (2n+1) "sobrando" (mas cuja raiz n-ésima tende a 1) para obter
(C_n)^{1/n} / 4 -> 1.

Abraços,
-- 
Bernardo Freitas Paulo da Costa

-- 
Esta mensagem foi verificada pelo sistema de antiv�rus e
 acredita-se estar livre de perigo.


=
Instru��es para entrar na lista, sair da lista e usar a lista em
http://www.mat.puc-rio.br/~obmlistas/obm-l.html
=


[obm-l] Re: [obm-l] Re: [obm-l] Re: [obm-l] Re: [obm-l] Determinante máximo

2015-08-25 Por tôpico Bernardo Freitas Paulo da Costa
2015-08-25 22:48 GMT-03:00 Rogerio Ponce abrlw...@gmail.com:
 Ola' Bernardo,

Oi Rogério.

 usando a mesma pintura de um tabuleiro de xadrez, temos a diagonal principal
 branca, e a diagonal secundaria preta.

 No caso dessa matriz 4x4, uma forma de se visualizar os termos que devem ser
 multiplicados entre si (para obtermos cada uma das 8 parcelas do
 determinante) e' a seguinte:

 a gente escreve 2 vezes a mesma matriz (uma ao lado da outra), e entao
 tomamos os 4 termos da diagonal principal, e tambem os 4 termos de mais 3
 linhas paralelas 'a diagonal principal (que, a exemplo do tabuleiro de
 xadrez, tambem vou chamar de diagonais);

 e depois executamos um procedimento semelhante com a diagonal secundaria,
 obtendo as 8 parcelas a serem somadas.

Esse procedimento prático para calcular determinantes só vale em
matrizes 3x3. Não vale em matrizes 2x2 (senão daria zero, como você
pode verificar) e em matrizes 4x4 isso dá apenas 8 termos, quando na
verdade há 4! = 24 termos no determinante.

Não sei porque ainda se ensina isto, porque é um caso muito
particular, e acaba gerando confusão quando se tenta ensinar o caso
geral. Além de ser muito menos poderoso e versátil do que a definição
por
- Recorrência (Lagrange, expansão em linhas ou colunas, bom quando tem
muitos zeros)
- Eliminação de Gauss (que é cúbico, e é melhor quando a matriz é cheia)

Abraços,
-- 
Bernardo Freitas Paulo da Costa

-- 
Esta mensagem foi verificada pelo sistema de antiv�rus e
 acredita-se estar livre de perigo.


=
Instru��es para entrar na lista, sair da lista e usar a lista em
http://www.mat.puc-rio.br/~obmlistas/obm-l.html
=


  1   2   3   4   5   6   7   >